Nclex Test 03

You might also like

Download as docx, pdf, or txt
Download as docx, pdf, or txt
You are on page 1of 57

NCLEX-RN Practice Quiz Test Bank #3 (75

Questions)
NCLEXRN-03-001

Question Tag: Parkinson’s disease

Question Category: Safe and Effective Care Environment, Management of Care 

A patient with Parkinson’s disease has a nursing diagnosis of Impaired Physical


Mobility related to neuromuscular impairment. You observe a nursing assistant performing
all of these actions. For which action must you intervene?

 A. The NA assists the patient to ambulate to the bathroom and back to bed.
 B. The NA reminds the patient not to look at his feet when he is walking.
 C. The NA performs the patient’s complete bath and oral care.
 D. The NA sets up the patient’s tray and encourages the patient to feed
himself.
Correct Answer: C. The NA performs the patient’s complete bath and oral care.

The nursing assistant should assist the patient with morning care as needed, but the goal is
to keep this patient as independent and mobile as possible.

Option A: Assisting the patient to ambulate prevents incidences of fall and



injury.
 Option B: Reminding the patient not to look at his feet while walking
maintains the client’s independence while keeping him safe.
 Option D: Encouraging the patient to feed himself is an appropriate goal of
maintaining independence. 
NCLEXRN-03-002

Question Tag: low back pain

Question Category: Health Promotion and Maintenance

The nurse is preparing to discharge a patient with chronic low back pain. Which statement
by the patient indicates that additional teaching is necessary?

 A. “I will avoid exercise because the pain gets worse.”


 B. “I will use heat or ice to help control the pain.”
 C. “I will not wear high-heeled shoes at home or work.”
 D. “I will purchase a firm mattress to replace my old one.”
Correct Answer: A. “I will avoid exercise because the pain gets worse.”

Exercises are used to strengthen the back, relieve pressure on compressed nerves and
protect the back from re-injury. Doing exercises to strengthen the lower back can help
alleviate and prevent lower back pain. It can also strengthen the core, leg, and arm
muscles. According to researchers, exercise also increases blood flow to the lower back
area, which may reduce stiffness and speed up the healing process.

Option B: Ice and heat application are appropriate interventions for back pain.

Applying ice or a reusable gel pack constricts blood vessels and reduces
swelling around the injury. This is particularly useful for conditions, like a
sprained ankle, that cause significant swelling. Heat has the opposite effect,
increasing blood flow to the area. This relaxes muscle fibers, which can help
when the client experiences spasms or stiffness.
 Option C: People with chronic back pain should avoid wearing high-heeled
shoes at all times. The normal s-curve of the spine acts as a cushion or spring,
reducing stress on the vertebrae. When wearing high heels, the shape of the
spine is altered and the client doesn’t get that same shock absorption as she
walks, which, over time, can lead to uneven wear on the cartilage discs, joints
and ligaments of the back.
 Option D: A firm mattress prevents lower back pain. Sleeping on a mattress
that is too firm can cause aches and pains on pressure points. A medium-firm
mattress may be more comfortable because it allows the shoulder and hips to
sink in slightly. Patients who want a firmer mattress for back support can get
one with thicker padding for greater comfort.
NCLEXRN-03-003

Question Tag: Spinal cord injury

Question Category: Physiological Integrity, Physiological Adaptation

A patient with a spinal cord injury (SCI) complains about a severe throbbing headache that
suddenly started a short time ago. Assessment of the patient reveals increased blood
pressure (168/94) and decreased heart rate (48/minute), diaphoresis, and flushing of the
face and neck. What action should you take first?

 A. Administer the ordered acetaminophen (Tylenol).


 B. Check the Foley tubing for kinks or obstruction.
 C. Adjust the temperature in the patient’s room.
 D. Notify the physician about the change in status.
Correct Answer: B. Check the Foley tubing for kinks or obstruction.

These signs and symptoms are characteristic of autonomic dysreflexia, a neurologic


emergency that must be promptly treated to prevent a hypertensive stroke. The cause of
this syndrome is noxious stimuli, most often a distended bladder or constipation, so
checking for poor catheter drainage, bladder distention, or fecal impaction is the first
action that should be taken.

Option C: Adjusting the room temperature may be helpful, since too cool a

temperature in the room may contribute to the problem.
 Option A: Tylenol will not decrease the autonomic dysreflexia that is causing
the patient’s headache.
 Option D: Notification of the physician may be necessary if nursing actions do
not resolve symptoms.
NCLEXRN-03-004

Question Tag: Neurologic unit

Question Category: Safe and Effective Care Environment, Management of Care

Which patient should you, as charge nurse, assign to a new graduate RN who is orienting
to the neurologic unit?

 A. A 28-year-old newly admitted patient with spinal cord injury.


 B. A 67-year-old patient with a stroke 3 days ago and left-sided weakness.
 C. An 85-year-old dementia patient to be transferred to long-term care today.
 D. A 54-year-old patient with Parkinson’s who needs assistance with bathing.
Correct Answer: B. A 67-year-old patient with stroke 3 days ago and left-sided
weakness.

The new graduate RN who is oriented to the unit should be assigned stable, non-complex
patients, such as the patient with stroke.

Option A: The newly admitted SCI should be assigned to experienced nurses.



Most cases of SCI take place when trauma breaks and squeezes the vertebrae,
or the bones of the back. This, in turn, damages the axons—the long nerve cell
“wires” that pass through vertebrae, carrying signals between the brain and
the rest of the body. The axons might be crushed or completely severed by
this damage. Someone with injury to only a few axons might be able to
recover completely from their injury. On the other hand, a person with
damage to all axons will most likely be paralyzed in the areas below the injury.
 Option C: A patient for transfer should be assigned to a nurse who has
experience in the process of transferring patients.
 Option D: The patient with Parkinson’s disease needs assistance with bathing,
which is best delegated to the nursing assistant.
NCLEXRN-03-005

Question Tag: Spinal cord injury


Question Category: Physiological Integrity, Physiological Adaptation

A patient with a spinal cord injury at level C3-4 is being cared for in the ED. What is the
priority assessment?

 A. Determine the level at which the patient has intact sensation.


 B. Assess the level at which the patient has retained mobility.
 C. Check blood pressure and pulse for signs of spinal shock.
 D. Monitor respiratory effort and oxygen saturation level.
Correct Answer: D. Monitor respiratory effort and oxygen saturation level.

The first priority for the patient with an SCI is assessing respiratory patterns and ensuring
an adequate airway. The patient with a high cervical injury is at risk for respiratory
compromise because the spinal nerves (C3 – 5) innervate the phrenic nerve, which controls
the diaphragm.

Option A: Determining this data can be done after addressing the concerns

on the respiratory status of the patient.
 Option B: This data can be assessed after monitoring the respiratory effort
and oxygen saturation level of the patient.
 Option C: Vital signs checking is also necessary, but not as high priority. Vital
signs can be quite abnormal following SCI. In addition to the usual causes in
trauma such as pain, bleeding and distress, this can be due to loss of
autonomic control, which occurs particularly in cervical or high thoracic
injuries. The autonomic nervous system controls our HR, BP temperature etc.
Autonomic instability is most acute in the first few days to weeks of the injury. 
NCLEXRN-03-006

Question Tag: delegation

Question Category: Safe and Effective Care Environment, Management of Care

You are pulled from the ED to the neurologic floor. Which action should you delegate to
the nursing assistant when providing nursing care for a patient with SCI?

 A. Assess the patient’s respiratory status every 4 hours.


 B. Take a patient’s vital signs and record them every 4 hours.
 C. Monitor nutritional status including calorie counts.
 D. Have the patient turn, cough, and deep breathe every 3 hours.
Correct Answer: B. Take a patient’s vital signs and record them every 4 hours.

The nursing assistant’s training and education include taking and recording a patient’s vital
signs.
Option A: Assessing the patient’s respiratory status would require the

knowledge of a registered nurse.
 Option C: Monitoring patients requires additional education and is
appropriate to the scope of practice for professional nurses.
 Option D: The nursing assistant may assist with turning and repositioning the
patient and may remind the patient to cough and deep breathe but does not
teach the patient how to perform these actions.
NCLEXRN-03-007

Question Tag: spinal cord injury

Question Category: Basic Care and Comfort

You are helping the patient with an SCI to establish a bladder-retraining program. What
strategies may stimulate the patient to void? Select all that apply.

 A. Stroke the patient’s inner thigh.


 B. Pull on the patient’s pubic hair.
 C. Initiate intermittent straight catheterization.
 D. Pour warm water over the perineum.
 E. Tap the bladder to stimulate detrusor muscle.
Correct Answers: A, B, D, and E

All of the strategies, except straight catheterization, may stimulate voiding in patients with
SCI.

Option C: Intermittent bladder catheterization can be used to empty the



patient’s bladder, but it will not stimulate voiding. 
NCLEXRN-03-008

Question Tag: cervical SCI

Question Category: Safe and Effective Care Environment, Management of Care

The patient with a cervical SCI has been placed in fixed skeletal traction with a halo fixation
device. When caring for this patient the nurse may delegate which action (s) to the
LPN/LVN?  Select all that apply.

 A. Check the patient’s skin for pressure from the device.


 B. Assess the patient’s neurologic status for changes.
 C. Observe the halo insertion sites for signs of infection.
 D. Clean the halo insertion sites with hydrogen peroxide.
Correct Answer: A, C, and D
Option A: Checking for signs of pressure within the scope of practice of the

LPN/LVN. 
 Option C: Observing for signs of an infection is within the scope of practice of
the LPN/LVN.
 Option D: The LPN/LVN also has the appropriate skills for cleaning the halo
insertion sites with hydrogen peroxide.
 Option B: Neurologic examination requires additional education and skill
appropriate to the professional RN.
NCLEXRN-03-009

Question Tag: SCI

Question Category: Physiological Integrity, Physiological Adaptation

You are preparing a nursing care plan for the patient with SCI including the nursing
diagnosis Impaired Physical Mobility and Self-Care Deficit. The patient tells you, “I don’t
know why we’re doing all this. My life’s over.” What additional nursing diagnosis
takes priority based on this statement?

 A. Risk for Injury related to altered mobility


 B. Imbalanced Nutrition, Less Than Body Requirements
 C. Impaired Adjustment to Spinal Cord Injury
 D. Poor Body Image related to immobilization
Correct Answer: C. Impaired Adjustment to Spinal Cord Injury

The patient’s statement indicates impairment of adjustment to the limitations of the injury
and indicates the need for additional counseling, teaching, and support.

Option A: The patient may have a risk for injury due to SCI, but it is not

related to the statement. The spinal cord can be injured by transection,
distraction, compression, bruising, haemorrhage or ischaemia of the cord or
by injury to blood vessels supplying it. These injuries can all result in
permanent cord injury and may be complete or incomplete.
 Option B: A client with SCI may have an imbalanced nutrition because of a
lesser appetite for eating, but it is not directly related to the statement. Refer
to Dietician early to ensure adequate nutrition, fluid & fibre in the feeds.
 Option D: This nursing diagnosis may be appropriate to the patient with SCI,
but it is not related to the patient’s statement.
NCLEXRN-03-010

Question Tag: neurologic care

Question Category: Safe and Effective Care Environment, Management of Care


Which patient should be assigned to the traveling nurse, new to neurologic nursing care,
who has been in the neurologic unit for 1 week?

 A. A 34-year-old patient newly diagnosed with multiple sclerosis (MS).


 B. A 68-year-old patient with chronic amyotrophic lateral sclerosis (ALS).
 C. A 56-year-old patient with Guillain-Barre syndrome (GBS) in respiratory
distress.
 D. A 25-year-old patient admitted with CA level spinal cord injury (SCI).
Correct Answer: B. A 68-year-old patient with chronic amyotrophic lateral sclerosis
(ALS)

The traveling nurse is relatively new to neurologic nursing and should be assigned to
patients whose conditions are stable and not complex. 

 Option A: The newly diagnosed patient will need to be transferred to the ICU.
Multiple sclerosis (MS) is an immune-mediated inflammatory disease that
attacks myelinated axons in the central nervous system, destroying the myelin
and the axon in variable degrees and producing significant physical disability
within 20–25 years in more than 30% of patients. The hallmark of MS is
symptomatic episodes that occur months or years apart and affect different
anatomic locations.
 Option C: The patient with GBS is in respiratory distress and should be
assigned to an experienced neurological nurse. Guillain-Barré syndrome (GBS)
is a rare disorder in which a person’s own immune system damages their nerve
cells, causing muscle weakness and sometimes paralysis. GBS can cause
symptoms that usually last for a few weeks.
 Option D: The patient with C4 SCI is at risk for respiratory arrest. A C4 spinal
cord injury occurs when damage is dealt about mid-way down the cervical
spinal cord — the topmost portion of the spinal cord that is located in the
neck and upper shoulders.

NCLEXRN-03-011

Question Tag: multiple sclerosis

Question Category: Physiological Integrity, Basic Care and Comfort

The patient with multiple sclerosis tells the nursing assistant that after physical therapy she
is too tired to take a bath. What is your priority nursing diagnosis at this time?

 A. Fatigue related to disease state


 B. Activity Intolerance due to generalized weakness
 C. Impaired Physical Mobility related to neuromuscular impairment
 D. Self-care Deficit related to fatigue and neuromuscular weakness
Correct Answer: D. Self-care Deficit related to fatigue and neuromuscular weakness

At this time, based on the patient’s statement, the priority is Self-Care Deficit related to
fatigue after physical therapy. Fatigue is described as an overwhelming feeling of lassitude
or lack of physical or mental energy that interferes with activities.

Option A: The patient might be experiencing fatigue, but it might be due to



the activities at physical therapy. Fatigue is one of the most common
symptoms of MS, reported by at least 75% of patients with the disease.
 Option B: Activity intolerance in a patient with MS is appropriate, but not
related to the statement. An estimated 50–60% of persons with MS describe
fatigue as one of their most bothersome symptoms, and it is a major reason
for unemployment among MS patients.
 Option C: Impaired physical mobility is appropriate to a patient with MS, but
it is not related to the patient’s statement. Spasticity in MS is characterized by
increased muscle tone and resistance to movement; it occurs most frequently
in muscles that function to maintain upright posture. The muscle stiffness
greatly increases the energy expended to perform activities of daily living
(ADLs), which in turn contributes to fatigue.
NCLEXRN-03-012

Question Tag: GBS

Question Category: Physiological Integrity, Reduction of Risk Potential

The LPN/LVN, under your supervision, is providing nursing care for a patient with GBS.
What observation would you instruct the LPN/LVN to report immediately?

 A. Complaints of numbness and tingling.


 B. Facial weakness and difficulty speaking.
 C. Rapid heart rate of 102 beats per minute.
 D. Shallow respirations and decreased breath sounds.
Correct Answer: D. Shallow respirations and decreased breath sounds

The priority interventions for the patient with GBS are aimed at maintaining adequate
respiratory function. These patients are at risk for respiratory failure, which is urgent. Upon
presentation, 40% of patients have respiratory or oropharyngeal weakness. Ventilatory
failure with required respiratory support occurs in up to one third of patients at some time
during the course of their disease.

 Option A: These findings should be reported to the nurse but it is not an


urgent matter. The typical patient with Guillain-Barré syndrome (GBS), which in
most cases will manifest as acute inflammatory demyelinating
polyradiculoneuropathy (AIDP), presents 2-4 weeks following a relatively
benign respiratory or gastrointestinal illness with complaints of finger
dysesthesias and proximal muscle weakness of the lower extremities.
 Option B: Facial weakness and difficulty of speaking are common signs of GBS
and must be reported, but it is not a priority. The classic clinical picture of
weakness is ascending and symmetrical in nature. The lower limbs are usually
involved before the upper limbs. Proximal muscles may be involved earlier
than the more distal ones. Trunk, bulbar, and respiratory muscles can be
affected as well.
 Option C: A rapid heart rate is important and should be reported to the nurse,
but it is not life-threatening. Autonomic nervous system involvement with
dysfunction in the sympathetic and parasympathetic systems can be observed
in patients with GBS.
NCLEXRN-03-013

Question Tag: myasthenia gravis

Question Category: Physiological Adaptation

The nursing assistant reports to you, the RN, that the patient with myasthenia gravis (MG)
has an elevated temperature (102.20 F), heart rate of 120/minute, rise in blood pressure
(158/94), and was incontinent of urine and stool. What is your best first action at this time?

 A. Administer an acetaminophen suppository


 B. Notify the physician immediately
 C. Recheck vital signs in 1 hour
 D. Reschedule patient’s physical therapy session
Correct Answer: B. Notify the physician immediately.

The changes that the nursing assistant is reporting are characteristics of myasthenia crisis,
which often follows some type of infection. The patient is at risk for inadequate respiratory
function. In addition to notifying the physician, the nurse should carefully monitor the
patient’s respiratory status. The patient may need intubation and mechanical ventilation.

Option A: The nurse would notify the physician before giving the suppository

because there may be orders for cultures before giving acetaminophen.
 Option C: This patient’s vital signs need to be re-checked sooner than 1 hour.
 Option D: Rescheduling the physical therapy can be delegated to the unit
clerk and is not urgent. Focus: Prioritization
NCLEXRN-03-014

Question Tag: acute hemorrhagic stroke

Question Category: Physiological Integrity, Pharmacological and Parenteral Therapies


You are providing care for a patient with an acute hemorrhage stroke. The patient’s
husband has been reading a lot about strokes and asks why his wife did not receive
alteplase. What is your best response?

 A. “Your wife was not admitted within the time frame that alteplase is usually
given.”
 B. “This drug is used primarily for patients who experience an acute heart
attack.”
 C. “Alteplase dissolves clots and may cause more bleeding into your wife’s
brain.”
 D. “Your wife had gallbladder surgery just 6 months ago and this prevents the
use of alteplase.”
Correct Answer: C. “Alteplase dissolves clots and may cause more bleeding into your
wife’s brain.”

Alteplase is a clot buster. With a patient who has experienced a hemorrhagic stroke, there
is already bleeding into the brain. A drug like alteplase can worsen the bleeding.

Option A: This statement is correct, but it does not address the relevance of

the drug to the patient’s diagnosis. 
 Option B: This is an accurate statement, but it does not clearly explain the
effects of the drug on the patient.
 Option D: This statement is also accurate about use of alteplase, but it is not
pertinent to this patient’s diagnosis.
NCLEXRN-03-015

Question Tag: right hemisphere stroke

Question Category: Safe and Effective Care Environment, Management of Care

You are supervising a senior nursing student who is caring for a patient with a right
hemisphere stroke. Which action by the student nurse requires that you intervene?

 A. The student instructs the patient to sit up straight, resulting in the patient’s
puzzled expression.
 B. The student moves the patient’s tray to the right side of her over-bed tray.
 C. The student assists the patient with passive range-of-motion (ROM)
exercises.
 D. The student combs the left side of the patient’s hair when the patient
combs only the right side.
Correct Answer: A. The student instructs the patient to sit up straight, resulting in the
patient’s puzzled expression.
Patients with right cerebral hemisphere stroke often present with neglect syndrome. They
lean to the left and when asked, respond that they believe they are sitting up straight. They
often neglect the left side of their bodies and ignore food on the left side of their food
trays. The nurse would need to remind the student of this phenomenon and discuss the
appropriate interventions. 

Option B: They often ignore the food on the left side of their food trays, so

reminding them is a correct action.
 Option C: Passive range of motion exercises help keep the left side of the
patient’s body from atrophy as a result of unuse.
 Option D: This is a correct action of the student nurse, as the patient may
neglect the left side of the body.
NCLEXRN-03-016

Question Tag: arteries

Question Category: Physiological Integrity, Physiological Adaptation

Which of the following arteries primarily feeds the anterior wall of the heart?

 A. Circumflex artery
 B. Internal mammary artery
 C. Left anterior descending artery
 D. Right coronary artery
Correct Answer: C. Left anterior descending artery

The left anterior descending artery is the primary source of blood for the anterior wall of
the heart. The left anterior descending artery branches off the left coronary artery and
supplies blood to the front of the left side of the heart.

Option A: The circumflex artery supplies the lateral wall. The circumflex artery,

fully titled as the circumflex branch of the left coronary artery, is an artery that
branches off from the left coronary artery to supply portions of the heart with
oxygenated blood. The circumflex artery itself divides into smaller arterial
systems.
 Option B: The internal mammary artery supplies the mammary. The internal
thoracic artery (ITA), previously commonly known as the internal mammary
artery (a name still common among surgeons), is an artery that supplies the
anterior chest wall and the breasts.
 Option D: The right coronary artery supplies the inferior wall of the heart. The
right coronary artery supplies blood to the right ventricle and then supplies
the underside (inferior wall) and backside (posterior wall) of the left ventricle.
NCLEXRN-03-017
Question Tag: coronary arteries

Question Category: Physiological Integrity, Physiological Adaptation

When do coronary arteries primarily receive blood flow?

 A. During inspiration
 B. During diastole
 C. During expiration
 D. During systole
Correct Answer: B. During diastole

Although the coronary arteries may receive a minute portion of blood during systole, most
of the blood flow to coronary arteries is supplied during diastole.

Option A: Breathing patterns are irrelevant to blood flow. It has been



suggested that the diaphragm will preferentially steal blood flow from working
locomotor muscles during increased activity (Bradley & Leith, 1978; Musch,
1993). In healthy adults, the cost of breathing is <5% of the total oxygen
consumption at low level exercise but approaches 15% during heavy exercise
in young athletes or older fit subjects (Aaron et al. 1992; Dempsey & Johnson,
1992). Further, a reflex vasoconstriction of the locomotor muscles is evident
when a substantial respiratory load is applied sufficient to elicit diaphragm
fatigue
 Option C: Expiration is not related to the blood flow. The pulmonary system is
intimately linked with the cardiovascular system anatomically and
hemodynamically and plays a significant role in exercise intolerance through a
number of mechanisms (Olson et al. 2006a,b;).
 Option D: There is a little portion of the blood that the coronary arteries
receive during systole. During systole, intramuscular blood vessels are
compressed and twisted by the contracting heart muscle and blood flow to
the left ventricle is at its lowest. The force is greatest in the subendocardial
layers where it approximates to intramyocardial pressure.
NCLEXRN-03-018

Question Tag: cause of death

Question Category: Health Promotion and Maintenance

Which of the following illnesses is the leading cause of death in the US?

 A. Cancer
 B. Coronary artery disease
 C. Liver failure
 D. Renal failure
Correct Answer: B. Coronary artery disease

Coronary artery disease accounts for over 50% of all deaths in the US.

Option A: Cancer accounts for approximately 20%.



 Option C: Liver failure accounts for less than 10% of all deaths in the US.
 Option D: Less than 10% of all deaths in the US can be attributed to renal
failure.
NCLEXRN-03-019

Question Tag: coronary artery disease

Question Category: Physiological Integrity, Physiological Adaptation

Which of the following conditions most commonly results in CAD?

 A. Atherosclerosis
 B. DM
 C. MI
 D. Renal failure
Correct Answer: A. Atherosclerosis

Atherosclerosis, or plaque formation, is the leading cause of CAD.

Option B: DM is a risk factor for CAD but isn’t the most common cause. Near-

normal glycemic control for a median of 3.5 to 5 years does not reduce
cardiovascular events. Thus, the general goal of HbA1c <7% appears
reasonable for the majority of patients. Iatrogenic hypoglycemia is the limiting
factor in the glycemic management of diabetes, and is an independent cause
of excess morbidity and mortality.
 Option D: Renal failure doesn’t cause CAD, but the two conditions are
related. Chronic kidney disease (CKD) accelerates the course of coronary artery
disease, independent of conventional cardiac risk factors. In addition, CKD has
been shown to confer inferior clinical outcomes following successful coronary
revascularisation, which may be offset by arterial grafting.
 Option C: Myocardial infarction is commonly a result of CAD. Myocardial
infarction occurs when a coronary artery is so severely blocked that there is a
significant reduction or break in the blood supply, causing damage or death to
a portion of the myocardium (heart muscle).
NCLEXRN-03-020
Question Tag: atherosclerosis

Question Category: Physiological Integrity, Physiological Adaptation

Atherosclerosis impedes coronary blood flow by which of the following mechanisms?

A. Plaques obstruct the vein


 B. Plaques obstruct the artery
 C. Blood clots form outside the vessel wall
 D. Hardened vessels dilate to allow the blood to flow through
Correct Answer: B. Plaques obstruct the artery.

Arteries, not veins, supply the coronary arteries with oxygen and other nutrients.
Atherosclerosis is a lipoprotein-driven disease that leads to plaque formation at specific
sites of the arterial tree through intimal inflammation, necrosis, fibrosis, and calcification.

Option A: Atherosclerosis is a direct result of plaque formation in the artery.



Most often, the culprit morphology is plaque rupture with exposure of highly
thrombogenic, red cell–rich necrotic core material. The permissive structural
requirement for this to occur is an extremely thin fibrous cap, and thus,
ruptures occur mainly among lesions defined as thin-cap fibroatheromas.
 Option C: Blood clots form inside the vessel wall and impede the circulation.
Also common are thrombi forming on lesions without rupture (plaque
erosion), most often on pathological intimal thickening or fibroatheromas.
However, the mechanisms involved in plaque erosion remain largely unknown,
although coronary spasm is suspected.
 Option D: Hardened vessels can’t dilate properly and, therefore, constrict
blood flow. During atherogenesis, the local vessel segment tends to remodel
in such a way that the lumen area is usually not compromised until plaques
are large (expansive remodeling). Thereafter stenosis formation may occur
through continued plaque growth or shrinkage of the local vessel segment
(constrictive remodeling) or a combination of the 2 processes.
NCLEXRN-03-021

Question Tag: coronary artery disease

Question Category: Health Promotion and Maintenance

Which of the following risk factors for coronary artery disease cannot be corrected?

 A. Cigarette smoking
 B. DM
 C. Heredity
 D. HPN
Correct Answer: C. Heredity

Because “heredity” refers to our genetic makeup, it can’t be changed.

Option A: Cigarette smoking cessation is a lifestyle change that involves



behavior modification. Smoking raises the risk of getting CAD and dying early
from CAD. Carbon monoxide, nicotine, and other substances in tobacco smoke
can promote atherosclerosis and trigger symptoms of coronary artery disease.
 Option B: Diabetes mellitus is a risk factor that can be controlled with diet,
exercise, and medication. Over time, high blood sugar can damage blood
vessels and the nerves that control the heart. People with diabetes are also
more likely to have other conditions that raise the risk for heart disease: High
blood pressure increases the force of blood through your arteries and can
damage artery walls.
 Option D: Altering one’s diet, exercise, and medication can correct
hypertension. British Hypertension Society (BHS) guidelines state that advice
should be provided for prevention as well as treatment of hypertension and
should be given to pre-hypertensives and those with a strong family history.
They point out that effective lifestyle modification can lower blood pressure by
at least as much as a single antihypertensive drug.
NCLEXRN-03-022

Question Tag: cholesterol, coronary artery disease

Question Category: Physiological Integrity, Reduction of Risk Potential

Exceeding which of the following serum cholesterol levels significantly increases the risk of
coronary artery disease?

 A. 100 mg/dl
 B. 150 mg/dl
 C. 175 mg/dl
 D. 200 mg/dl
Correct Answer: D. 200 mg/dl

Cholesterol levels above 200 mg/dl are considered excessive. They require dietary
restriction and perhaps medication. Exercise also helps reduce cholesterol levels. The other
levels listed are all below the nationally accepted levels for cholesterol and carry a lesser
risk for CAD. Normal levels of serum cholesterol is within 125 to 200 mg/dl.
Option A: 100 mg/dl is an acceptable level of serum cholesterol. An elevated

low-density lipoprotein cholesterol (LDL-C) level is a major risk factor for CAD,
and several large, randomized, primary prevention trials have shown that
lowering LDL-C levels with statins reduces the risk of major coronary events
and coronary death.
 Option B: 150 mg/dl is within the normal level of serum cholesterol. LDL is the
particle that is responsible for transporting cholesterol to tissues. Cholesterol
transportation is achieved by binding of the LDL receptor and apoB.
 Option C: 175 mg/dl is still an acceptable level of serum cholesterol. HDL is a
molecule that is antioxidant, antiinflammatory, antiapoptotic and increases
macrophage cholesterol excretion and endothelial healing. The removal of
cholesterol from the body by the liver via HDL is called reverse cholesterol
transport.
NCLEXRN-03-023

Question Tag: coronary artery disease

Question Category: Physiological Integrity, Physiological Adaptation

Which of the following actions is the first priority care for a client exhibiting signs and
symptoms of coronary artery disease?

 A. Decrease anxiety
 B. Enhance myocardial oxygenation
 C. Administer sublingual nitroglycerin
 D. Educate the client about his symptoms
Correct Answer: B. Enhance myocardial oxygenation

Enhancing myocardial oxygenation is always the first priority when a client exhibits signs
and symptoms of cardiac compromise. Without adequate oxygen, the myocardium suffers
damage.

 Option A: Decreasing the client’s anxiety is also important, but it is not the
priority. When someone is anxious, their body reacts in ways that can put an
extra strain on their heart. The physical symptoms of anxiety can be especially
damaging among individuals with existing cardiac disease.
 Option C: Sublingual nitroglycerin is administered to treat acute angina, but
its administration isn’t the first priority. Although nitroglycerin has a
vasodilatory effect in both arteries and veins, the profound desired effects
caused by nitroglycerin are primarily due to venodilation. Venodilation causes
pooling of blood within the venous system, reducing preload to the heart,
which causes a decrease in cardiac work, reducing anginal symptoms
secondary to demand ischemia. 
Option D: Although educating the client is important in care delivery, it is not

a priority when a client is compromised. Patient education promotes patient-
centered care and increases adherence to medication and treatments. An
increase in compliance leads to a more efficient and cost-effective healthcare
delivery system. Educating patients ensures continuity of care and reduces
complications related to the illness.
NCLEXRN-03-024

Question Tag: coronary artery disease

Question Category: Physiological Integrity, Physiological Adaptation

Medical treatment of coronary artery disease includes which of the following procedures?

 A. Cardiac catheterization
 B. Coronary artery bypass surgery
 C. Oral medication administration
 D. Percutaneous transluminal coronary angioplasty
Correct Answer: C. Oral medication administration.

Oral medication administration is a noninvasive, medical treatment for coronary artery


disease. Nitroglycerin is the most common vasodilator used for acute cases of angina. It
works to dilate or widen the coronary arteries, increasing blood flow to the heart muscle
and to relax the veins, lessening the amount of blood that returns to the heart from the
body. This combination of effects decreases the amount of work for the heart.

Option A: Cardiac catheterization isn’t a treatment but a diagnostic tool.



Cardiac catheterization (cardiac cath or heart cath) is a procedure to examine
how well the heart is working. A thin, hollow tube called a catheter is inserted
into a large blood vessel that leads to the heart. 
 Option B: Coronary artery bypass surgery is an invasive procedure. Coronary
artery bypass grafting (CABG) is a procedure to improve poor blood flow to
the heart. It may be needed when the arteries supplying blood to heart tissue,
called coronary arteries, are narrowed or blocked. This surgery may lower the
risk of serious complications for people who have obstructive coronary artery
disease, a type of ischemic heart disease. CABG may also be used in an
emergency, such as a severe heart attack.
 Option D: Percutaneous transluminal coronary angioplasty is an invasive,
surgical treatment. Percutaneous transluminal coronary angioplasty (PTCA) is a
minimally invasive procedure to open up blocked coronary arteries, allowing
blood to circulate unobstructed to the heart muscle.
NCLEXRN-03-025

Question Tag: infarction


Question Category: Physiological Integrity, Physiological Adaptation

Prolonged occlusion of the right coronary artery produces an infarction in which of the
following areas of the heart?

 A. Anterior
 B. Apical
 C. Inferior
 D. Lateral
Correct Answer: C. Inferior

The right coronary artery supplies the right ventricle or the inferior portion of the heart.
Therefore, prolonged occlusion could produce an infarction in that area.

Option A: The right coronary artery doesn’t supply the anterior portion (left

ventricle) of the heart. The right coronary artery supplies blood to the right
ventricle, the right atrium, and the SA (sinoatrial) and AV (atrioventricular)
nodes, which regulate the heart rhythm. The right coronary artery divides into
smaller branches, including the right posterior descending artery and the
acute marginal artery.
 Option B: The right coronary artery does not supply to the apical portion or
left ventricle of the heart. Together with the left anterior descending artery, the
right coronary artery helps supply blood to the middle or septum of the heart.
 Option D: The right coronary artery does not supply to the lateral portion
(some of the left ventricle and the left atrium) of the heart. The left main
coronary artery supplies blood to the left side of the heart muscle (the left
ventricle and left atrium).
NCLEXRN-03-026

Question Tag: myocardial infarction

Question Category: Physiological Integrity, Physiological Adaptation

Which of the following is the most common symptom of myocardial infarction?

 A. Chest pain
 B. Dyspnea
 C. Edema
 D. Palpitations
Correct Answer: A. Chest pain
The most common symptom of an MI is chest pain, resulting from deprivation of oxygen to
the heart. The classic manifestation of ischemia is usually described as a heavy chest
pressure or squeezing, a “burning” feeling, or difficulty in breathing. The discomfort or pain
often radiates to the left shoulder, neck, or arm. Chest pain may be atypical in a few cases.
It builds in intensity over a period of a few minutes. 

Option B: Dyspnea is the second most common symptom, related to an



increase in the metabolic needs of the body during an MI. Despite variable
prevalence estimates, dyspnea has been consistently associated with greater
mortality in the general population. It is a more powerful predictor of clinical
outcomes than objective physiologic measures such as pulmonary function
testing in the general population, or angina in patients referred for cardiac
evaluation.
 Option C: Edema is a later sign of heart failure, often seen after an MI. All the
factors which contribute to increased pressure in the left side and pooling of
blood on the left side of the heart can cause cardiogenic pulmonary edema.
The result of all these conditions will be increased pressure on the left side of
the heart: increased pulmonary venous pressure–> increased capillary pressure
in lungs–> pulmonary edema.
 Option D: Palpitations may result from reduced cardiac output, producing
arrhythmias. In patients who describe the palpitations as a brief flip-flopping
in the chest, the palpitations are thought to be caused by extra systoles such
as supraventricular or ventricular premature contractions. The flip-flop
sensation is thought to result from the forceful contraction following the
pause and the sensation that the heart is stopped results from the pause. The
sensation of rapid fluttering in the chest is thought to result from a sustained
ventricular or supraventricular arrhythmia. 
NCLEXRN-03-027

Question Tag: apical pulse

Question Category: Physiological Integrity, Physiological Adaptation

Which of the following landmarks is the correct one for obtaining an apical pulse?

 A. Left intercostal space, midaxillary line


 B. Left fifth intercostal space, midclavicular line
 C. Left second intercostal space, midclavicular line
 D. Left seventh intercostal space, midclavicular line
Correct Answer: B. Left fifth intercostal space, midclavicular line
The correct landmark for obtaining an apical pulse is the left intercostal space in the
midclavicular line. This is the point of maximum impulse and the location of the left
ventricular apex. 

Option A: Normally, heart sounds aren’t heard in the midaxillary line. During

systolic contraction of the heart, a high amplitude wave of blood gets ejected
through the aortic valve out towards the periphery.  This high-pressure wave
distends the arteries, especially compliant “elastic” or “conducting” arteries,
which tend to be larger and closer to the heart.  The subsequent release of
that distention somewhat sustains the systolic wave of blood throughout the
body, creating a spike followed by a downward sloping plateau in pulse
waveform. 
 Option C: The left second intercostal space in the midclavicular line is where
the pulmonic sounds are auscultated. The intensity of the pulse is determined
by blood pressure as well as other physiological factors such as ambient
temperature. For example, colder temperatures cause vasoconstriction leading
to decreased intensity. Besides the normal variation in a rhythm that occurs
with the respiratory cycle, the heart rate should be regular in the absence of
pathology. 
 Option D: Heart sounds are not heard in the seventh intercostal space in the
midclavicular line. Pulses are accurately measured when the clinician places
their fingertips onto the skin overlying the vessel (locations, see below) and
focuses on different aspects of the pulse. (NB: although one often hears that
utilization of the thumb for measuring pulses is less accurate secondary to
increased perception of the clinician’s own pulsation during palpation, the
author could not find data to support or refute this claim). If possible, the limb
under evaluation should have support throughout palpation.  
NCLEXRN-03-028

Question Tag: pain

Question Category: Physiological Integrity, Physiological Adaptation

Which of the following systems is the most likely origin of pain the client describes as
knifelike chest pain that increases in intensity with inspiration?

 A. Cardiac
 B. Gastrointestinal
 C. Musculoskeletal
 D. Pulmonary
Correct Answer: D. Pulmonary
Pulmonary pain is generally described by these symptoms. Pain may originate from several
different structures within the chest, including the skin, ribs, intercostal muscles, pleura,
esophagus, heart, aorta, diaphragm, or thoracic vertebrae. The pain may be transmitted by
intercostal, sympathetic, vagus, and phrenic nerves. The innervations of the deep structures
of the thorax follow common pathways to the central nervous system, making it difficult to
localize the source of pain. 

Option A: In cardiac pain, respirations are usually unaffected. Various schemes

have been used to classify the etiologies of chest pain, but the most useful is
to distinguish between acute and chronic patterns of pain. Patients with acute
pain include those whose episodes are of recent onset or those who have had
a recent increase in the intensity or frequency of recurrent pain. Patients
with chronic pain include those who have recurrent episodes of pain occurring
in a relatively stable pattern.
 Option B: GI pains don’t change with respiration. The findings suggest that
pain influences respiration by increasing its flow, frequency, and volume.
Furthermore, paced slow breathing is associated with pain reduction in some
of the studies, but evidence elucidating the underlying physiological
mechanisms of this effect is lacking.
 Option C: Musculoskeletal pain only increases with movement. The SNS is
concerned with the regulation of vascular tone, blood flow and blood
pressure, as sympathetic nerves have stimulating effects on the heart
(improving circulation) and respiratory system (increasing oxygen intake). Pain
therefore increases heart rate, blood pressure and respiratory rate.
NCLEXRN-03-029

Question Tag: heart murmurs

Question Category: Physiological Integrity, Physiological Adaptation

A murmur is heard at the second left intercostal space along the left sternal border. Which
valve area is this?

 A. Aortic
 B. Mitral
 C. Pulmonic
 D. Tricuspid
Correct Answer: C. Pulmonic

Abnormalities of the pulmonic valve are auscultated at the second left intercostal space
along the left sternal border. Murmurs of the cardiac system develop due to alterations in
blood flow or mechanical operation.  Murmurs develop from a multitude of mechanisms.
Typical cases include low blood viscosity from anemia, septal defects, failure of the ductus
arteriosus to close in newborns, excessive hydrostatic pressure on cardiac valves causing
valve failure, hypertrophic obstructive cardiomyopathy, and valvular specific pathologies. 

Option A: Aortic valve abnormalities are heard at the second intercostal



space, to the right of the sternum. Aortic regurgitation, also known as aortic
insufficiency, is a decrescendo blowing diastolic murmur heard best at the left
lower sternal border, heard when blood flows retrograde into the left ventricle.
This is most commonly seen in aortic root dilation and as sequelae of aortic
stenosis.
 Option B: Mitral valve abnormalities are heard at the fifth intercostal space in
the midclavicular line. Mitral stenosis is a diastolic murmur, best heard at the
left 5th midclavicular line. It is associated with infective endocarditis and
chronic rheumatic heart disease. Mitral regurgitation is a systolic murmur, best
heard at the left 5th midclavicular line with possible radiation to the left axilla.
It is commonly associated with infective endocarditis, rheumatic heart disease,
congenital anomalies, and inferior wall myocardial infarctions.
 Option D: Tricuspid valve abnormalities are heard at the third and fourth
intercostal spaces along the sternal border. Tricuspid stenosis is best heard at
the lower left sternal border. Typical causes include infective endocarditis, seen
in intravenous drug users, and carcinoid syndrome. Prolonged tricuspid
stenosis may lead to right atrial enlargement and arrhythmias. Tricuspid
regurgitation is systolic, auscultated at the lower left sternal border. It is also
associated with intravenous drug users and carcinoid syndrome.
NCLEXRN-03-030

Question Tag: cardiac damage

Question Category: Physiological Integrity, Reduction of Risk Potential

Which of the following blood tests is most indicative of cardiac damage?

 A. Lactate dehydrogenase
 B. Complete blood count
 C. Troponin I
 D. Creatine kinase
Correct Answer: C. Troponin I

Troponin I levels rise rapidly and are detectable within 1 hour of myocardial injury.
Troponin I levels aren’t detectable in people without cardiac injury. The troponin complex
consists of three subunits—troponin C, troponin I, and troponin T—and is located on the
myofibrillar thin (actin) filament of striated (skeletal and cardiac) muscle. The cardiac
isoforms troponin T and I are only expressed in cardiac muscle. Hence, cardiac troponin T
(cTnT) and I (cTnI) are more specific than creatine kinase (CK) values for myocardial injury
and, because of their high sensitivity, they may even be elevated when CK‐MB
concentrations are not. 

Option A: Lactate dehydrogenase is present in almost all body tissues and not

specific to heart muscle. LDH isoenzymes are useful in diagnosing cardiac
injury. The function of the enzyme is to catalyze the reversible conversion of
lactate to pyruvate with the reduction of NAD+ to NADH and vice versa.
 Option B: CBC is obtained to review blood counts, and a complete chemistry
is obtained to review electrolytes. Obtain a complete blood cell (CBC) count if
myocardial infarction (MI) is suspected in order to rule out anemia as a cause
of decreased oxygen supply and prior to giving thrombolytic agents.
 Option D: Because CK levels may rise with skeletal muscle injury, CK
isoenzymes are required to detect cardiac injury. In their current guidelines
from 2000, the Joint European Society of Cardiology/American College of
Cardiology committee redefined AMI as an elevation of cTn in blood above
the 99th centile of a healthy reference population in conjunction with signs or
symptoms of ischaemia. This did expand the diagnostic capacity to detect
micro‐MI which was not evident by CK‐MB measurements. 
NCLEXRN-03-031

Question Tag: morphine, myocardial infarction

Question Category: Physiological Integrity, Pharmacological and Parenteral therapies

What is the primary reason for administering morphine to a client with myocardial
infarction?

 A. To sedate the client


 B. To decrease the client’s pain
 C. To decrease the client’s anxiety
 D. To decrease oxygen demand on the client’s heart
Correct Answer: D. To decrease oxygen demand on the client’s heart

Morphine is administered because it decreases myocardial oxygen demand. Morphine to


relieve pain during a myocardial infarction (MI) has been in use since the early 1900s. In
2005, an observational study raised some concerns, but there are very few effective
alternatives. Morphine is a potent opioid; it decreases pain, which in turn leads to a
decrease in the activation of the autonomic nervous system. These are desirable effects
when a patient is having an MI.

 Option A: Morphine can also cause sedation on the client, but it is not the
main purpose of administering it. Morphine is rarely used for procedural
sedation. However, for small procedures, physicians will sometimes combine a
low dose of morphine with a low dose of benzodiazepine-like lorazepam.
 Option B: Pain is decreased when morphine is given, but it is not the primary
reason for administration. FDA-approved usage of morphine sulfate includes
moderate to severe pain that may be acute or chronic. Most commonly used
in pain management, morphine provides significant relief to patients afflicted
with pain.
 Option C: Morphine will also decrease anxiety, but isn’t primarily given for this
reason. Morphine can decrease the heart rate, blood pressure, and venous
return. Morphine can also stimulate local histamine-mediated processes.
NCLEXRN-03-032

Question Tag: myocardial infarction

Question Category: Physiological Integrity, Physiological Adaptation

Which of the following conditions is most commonly responsible for myocardial infarction?

 A. Aneurysm
 B. Heart failure
 C. Coronary artery thrombosis
 D. Renal failure
Correct Answer: C. Coronary artery thrombosis

Coronary artery thrombosis causes occlusion of the artery, leading to myocardial death.
Myocardial infarction occurs when a coronary artery is so severely blocked that there is a
significant reduction or break in the blood supply, causing damage or death to a portion of
the myocardium (heart muscle).

 Option A: An aneurysm is an outpouching of a vessel and doesn’t cause an


MI. On exertion, an elevated blood pressure could cause  expansion of the
aneurysmal cavity, aggravating the coronary ischemia, and eventually would
have produced myocardial infarction. However, this only occurs in very rare
cases.
 Option D: Renal failure can be associated with MI but isn’t a direct cause. All-
cause mortality of dialysis patients with acute myocardial infarction (AMI) is
59% at 1 year and about 73% at 2 years. AMI in patients with nondialysis-
dependent advanced CKD is also associated with poor long-term
cardiovascular outcomes and survival. 
 Option B: Heart failure is usually the result of an MI. It produces both
vasculopathy and left ventricular dysfunction and fibrosis. Endothelial
dysfunction in the coronary arteries can lead to acute coronary events. Left
ventricular dysfunction will cause the progression of heart failure, and left
ventricular fibrosis and dysfunction provide an arrhythmic substrate.
NCLEXRN-03-033

Question Tag: furosemide

Question Category: Physiological Integrity, Pharmacological and Parenteral therapies

What supplemental medication is most frequently ordered in conjunction with furosemide


(Lasix)?

 A. Chloride
 B. Digoxin
 C. Potassium
 D. Sodium
Correct Answer: C. Potassium

Supplemental potassium is given with furosemide because of the potassium loss that
occurs as a result of this diuretic. Loop diuretics act at the ascending loop of Henle in the
kidney and help the body push out extra fluid that could accumulate in the lungs or legs
and ankles when the heart is unable to completely pump blood throughout the body. But
they may also cause the body to eliminate excessive amounts of potassium, which might
be expected to increase mortality from heart arrhythmias. As a precaution therefore, many
doctors prescribe potassium supplements to their patients receiving loop diuretics.

Option A: Chloride isn’t lost during diuresis. Continued use of diuretics will

cause some overall sodium and chloride loss. The body, however, has a natural
way of compensating for these losses by reducing the excretion of sodium and
chloride and stabilizing the amount of sodium, chloride and water in the body.
In this manner, fluid depletion usually is prevented.
 Option B: Digoxin acts to increase contractility but isn’t given routinely with
furosemide. People with heart failure who take digoxin are commonly given
medicines called diuretics. These drugs remove excess fluid from the body.
Many diuretics can cause potassium loss. A low level of potassium in the body
can increase the risk of digitalis toxicity.
 Option D: Sodium is not lost during diuresis. Diuretic drugs increase urine
output by the kidney (i.e., promote diuresis). This is accomplished by altering
how the kidney handles sodium. If the kidney excretes more sodium, then
water excretion will also increase.
NCLEXRN-03-034

Question Tag: glucose, fatty acids


Question Category: Physiological Integrity, Physiological Adaptation

After myocardial infarction, serum glucose levels and free fatty acids are both increased.
What type of physiologic changes are these?

 A. Electrophysiologic
 B. Hematologic
 C. Mechanical
 D. Metabolic
Correct Answer: D. Metabolic

Both glucose and fatty acids are metabolites whose levels increase after a myocardial
infarction. Acute myocardial infarction evokes a characteristic neurohumoral response:
Catecholamine release is increased, plasma contents of free fatty acids and glucose are
elevated and glucose tolerance is diminished. 

Option A: Mechanical changes are those that affect the pumping action of the

heart. The main mechanical complications (MC) of acute myocardial infarction
are ventricular septal rupture(VSR), free wall rupture (FWR), and ischemic
mitral regurgitation (IMR). In the chronic phase, negative remodeling and
aneurysm formation may occur.
 Option B: Hematologic changes would affect the blood. It has been
documented that WBC associates through coronary atherosclerosis and ESR in
myocardial infarction. Variations occur in hematological parameters such as
hemoglobin, WBC, ESR, and platelet sedimentation rate and fibrinogen in
acute myocardial infarction.
 Option C: Electrophysiologic changes affect conduction. When coronary
blood flow is inadequate to support the oxygen needs of the myocardium (i.e.,
an ischemic state), tissue levels of oxygen fall, which leads to cellular hypoxia.
Severe and prolonged hypoxia can ultimately lead to cellular death and total
loss of electrical activity. Less severe hypoxia, or hypoxia of relatively short
duration, will produce electrophysiological (and mechanical) changes in the
heart.
NCLEXRN-03-035

Question Tag: heart sounds

Question Category: Physiological Integrity, Physiological Adaptation

Which of the following complications is indicated by a third heart sound (S3)?

 A. Ventricular dilation
 B. Systemic hypertension
 C. Aortic valve malfunction
 D. Increased atrial contractions
Correct Answer: A. Ventricular dilation

Rapid filling of the ventricles causes vasodilation that is auscultated as S3. The third heart
sound (S3) is a low-frequency, brief vibration occurring in early diastole at the end of the
rapid diastolic filling period of the right or left ventricle 

Option B: Systemic hypertension can result in a fourth heart sound. The fourth

heart sound is a low-pitched sound coincident with late diastolic filling of the
ventricle due to atrial contraction. It thus occurs shortly before the first heart
sound. Although it is also called the atrial sound, and its production requires
an effective atrial contraction, the fourth heart sound is the result of vibrations
generated within the ventricle.
 Option C: Aortic valve malfunction is heard as a murmur. One in three elderly
people have a heart murmur because of the scarring, thickening, or stiffening
of their aortic valve. That’s aortic sclerosis. It’s usually not dangerous, since the
valve can work for years after the murmur starts.
 Option D: Increased atrial contractions can cause a fourth heart sound.
Common theoretic mechanisms for this condition are based around abnormal
automaticity of the cardiac myocytes, increased eliciting events from chemical
or physical instigators, and reentry of a retrograde impulse. For these causes,
structural heart damage or chemical use may be found during the history and
physical examination.
NCLEXRN-03-036

Question Tag: crackles

Question Category: Physiological Integrity, Physiological Adaptation

After an anterior wall myocardial infarction, which of the following problems is indicated by
auscultation of crackles in the lungs?

 A. Left-sided heart failure


 B. Pulmonic valve malfunction
 C. Right-sided heart failure
 D. Tricuspid valve malfunction
Correct Answer: A. Left-sided heart failure

The left ventricle is responsible for the most cardiac output. An anterior wall MI may result
in a decrease in left ventricular function. When the left ventricle doesn’t function properly,
resulting in left-sided heart failure, fluid accumulates in the interstitial and alveolar spaces
in the lungs and causes crackles.

Option B: Pulmonic valve malfunction may cause right-sided heart failure.



Pulmonary regurgitation (PR, also called pulmonic regurgitation) is a leaky
pulmonary valve. This valve helps control the flow of blood passing from the
right ventricle to the lungs. A leaky pulmonary valve allows blood to flow back
into the heart chamber before it gets to the lungs for oxygen.
 Option C: Right-sided heart failure can be caused by pulmonic valve or
tricuspid valve malfunction. This is narrowing of the pulmonic valve that limits
blood flow out of the right ventricle.
 Option D: Tricuspid valve malfunction causes right-sided heart failure. The
tricuspid valve doesn’t close properly. This causes blood in the right ventricle
to flow back into the right atrium.
NCLEXRN-03-037

Question Tag: myocardial damage

Question Category: Physiological Integrity, Reduction of Risk Potential

Which of the following diagnostic tools is most commonly used to determine the location
of myocardial damage?

A. Cardiac catheterization


 B. Cardiac enzymes
 C. Echocardiogram
 D. Electrocardiogram
Correct Answer: D. Electrocardiogram

The ECG is the quickest, most accurate, and most widely used tool to determine the
location of myocardial infarction.

Option A: Cardiac catheterization is an invasive study for determining



coronary artery disease and may also indicate the location of myocardial
damage, but the study may not be performed immediately.
 Option B: Cardiac enzymes are used to diagnose MI but can’t determine the
location.
 Option C: An echocardiogram is used most widely to view myocardial wall
function after an MI has been diagnosed.
NCLEXRN-03-038

Question Tag: myocardial infarction


Question Category: Physiological Integrity, Physiological Adaptation

What is the first intervention for a client experiencing myocardial infarction?

 A. Administer morphine
 B. Administer oxygen
 C. Administer sublingual nitroglycerin
 D. Obtain an electrocardiogram
Correct Answer: B. Administer oxygen

Administering supplemental oxygen to the client is the first priority of care. The
myocardium is deprived of oxygen during an infarction, so additional oxygen is
administered to assist in oxygenation and prevent further damage.

Option A: Morphine may be given after administering supplemental oxygen



first. As a potent opioid, morphine has seemed to be the ideal analgesic. It has
innate hemodynamic effects that are beneficial during MI. It decreases heart
rate, blood pressure, and venous return, and it may also stimulate local
histamine-mediated processes. Theoretically, this reduces myocardial oxygen
demand.
 Option C: Sublingual nitroglycerin is also used to treat MI, but they’re more
commonly administered after the oxygen. Nitroglycerin remains a first-line
treatment for angina pectoris and acute myocardial infarction. Nitroglycerin
achieves its benefit by giving rise to nitric oxide, which causes vasodilation and
increases blood flow to the myocardium.
 Option D: An ECG is the most common diagnostic tool used to evaluate MI. In
a myocardial infarction transmural ischemia develops. In the first hours and
days after the onset of a myocardial infarction, several changes can be
observed on the ECG. First, large peaked T waves (or hyperacute T waves),
then ST elevation, then negative T waves and finally pathologic Q waves
develop.
NCLEXRN-03-039

Question Tag: myocardial infarction

Question Category: Psychosocial Integrity

What is the most appropriate nursing response to a myocardial infarction client who is


fearful of dying?

 A. “Tell me about your feelings right now.”


 B. “When the doctor arrives, everything will be fine.”
 C. “This is a bad situation, but you’ll feel better soon.”
 D. “Please be assured we’re doing everything we can to make you feel better.”
Correct Answer: A. “Tell me about your feelings right now.”

Validation of the client’s feelings is the most appropriate response. It gives the client a
feeling of comfort and safety.

Option B: Option B may give the client false hope.



 Option C: Telling the client that he will feel better soon would give him false
hope.
 Option D: No one can determine if a client experiencing MI will feel or get
better and therefore, these responses are inappropriate.
NCLEXRN-03-040

Question Tag: ischemic myocardium

Question Category: Physiological Integrity, Pharmacological and Parenteral therapies

Which of the following classes of medications protects the ischemic myocardium by


blocking catecholamines and sympathetic nerve stimulation?

 A. Beta-adrenergic blockers
 B. Calcium channel blockers
 C. Narcotics
 D. Nitrates
Correct Answer: A. Beta-adrenergic blockers

Beta-adrenergic blockers work by blocking beta receptors in the myocardium, reducing the
response to catecholamines and sympathetic nerve stimulation. They protect the
myocardium, helping to reduce the risk of another infarction by decreasing the workload of
the heart and decreasing myocardial oxygen demand.

Option B: Calcium channel blockers reduce the workload of the heart by



decreasing the heart rate.
 Option C: Narcotics reduce myocardial oxygen demand, promote
vasodilation, and decrease anxiety.
 Option D: Nitrates reduce myocardial oxygen consumption by decreasing left
ventricular end-diastolic pressure (preload) and systemic vascular resistance
(afterload).
NCLEXRN-03-041

Question Tag: myocardial infarction


Question Category: Physiological Integrity, Physiological Adaptation

What is the most common complication of a myocardial infarction?

 A. Cardiogenic shock
 B. Heart failure
 C. Arrhythmias
 D. Pericarditis
Correct Answer: C. Arrhythmias

Arrhythmias, caused by oxygen deprivation to the myocardium, are the most common
complication of an MI.

Option A: Cardiogenic shock, another complication of MI, is defined as the



end stage of left ventricular dysfunction. The condition occurs in
approximately 15% of clients with MI.
 Option B: Because the pumping function of the heart is compromised by an
MI, heart failure is the second most common complication.
 Option D: Pericarditis most commonly results from a bacterial or viral
infection but may occur after MI.
NCLEXRN-03-042

Question Tag: jugular vein distention

Question Category: Physiological Integrity, Physiological Adaptation

With which of the following disorders is jugular vein distention most prominent?

 A. Abdominal aortic aneurysm


 B. Heart failure
 C. Myocardial infarction
 D. Pneumothorax
Correct Answer: B. Heart failure

Elevated venous pressure, exhibited as jugular vein distention, indicates a failure of the
heart to pump.

 Option A: Jugular vein distention isn’t a symptom of abdominal aortic


aneurysm.  The jugular vein is considered a central vein in the body. Central
veins are thin-walled, distensible reservoirs and act as a conduit of blood in
continuity with the right atrium. The jugular vein divides into external and
internal.
Option C: An MI, if severe enough, can progress to heart failure; however, in

and of itself, an MI doesn’t cause jugular vein distention. In patients with acute
inferior-wall MI with right ventricular involvement, distention of neck veins is
commonly described as a sign of failure of the right ventricle. 
 Option D: Pneumothorax does not cause jugular vein distention. A tension
pneumothorax can cause severe hypotension (obstructive shock) and even
death. An increase in central venous pressure can result in distended neck
veins, hypotension. 
NCLEXRN-03-043

Question Tag: jugular vein distention

Question Category: Physiological Integrity, Physiological Adaptation

What position should the nurse place the head of the bed in to obtain the most accurate
reading of jugular vein distention?

 A. High-Fowler’s
 B. Raised 10 degrees
 C. Raised 30 degrees
 D. Supine position
Correct Answer: C. Raised 30 degrees

Jugular venous pressure is measured with a centimeter ruler to obtain the vertical distance
between the sternal angle and the point of highest pulsation with the head of the bed
inclined between 15 and 30 degrees.

Option A: In high-Fowler’s position, the veins would be barely discernible



above the clavicle.
 Option B: Inclined pressure can’t be seen when the head of the bed is raised
10 degrees because the point that marks the pressure level is above the jaw
(therefore, not visible).
 Option D: Inclined pressure cannot be seen when the client is in supine
position because the point that marks the pressure level is above the jaw.
NCLEXRN-03-044

Question Tag: digoxin

Question Category: Physiological Integrity, Pharmacological and Parenteral therapies

Which of the following parameters should be checked before administering digoxin?


 A. Apical pulse
 B. Blood pressure
 C. Radial pulse
 D. Respiratory rate
Correct Answer: A. Apical pulse

An apical pulse is essential for accurately assessing the client’s heart rate before
administering digoxin. The apical pulse is the most accurate point in the body.

Option B: Blood pressure is usually only affected if the heart rate is too low, in

which case the nurse would withhold digoxin.
 Option C: The radial pulse can be affected by cardiac and vascular disease and
therefore, won’t always accurately depict the heart rate.
 Option D: Digoxin has no effect on respiratory function. Digoxin is used to
treat heart failure, usually along with other medications. It is also used to treat
a certain type of irregular heartbeat (chronic atrial fibrillation).
NCLEXRN-03-045

Question Tag: toxicity

Question Category: Physiological Integrity, Pharmacological and Parenteral therapies

Toxicity from which of the following medications may cause a client to see a green halo
around lights?

 A. Digoxin
 B. Furosemide
 C. Metoprolol
 D. Enalapril
Correct Answer: A. Digoxin

One of the most common signs of digoxin toxicity is the visual disturbance known as the
green halo sign.

 Option B: Furosemide does not cause this kind of toxicity. The principal signs
and symptoms of overdose with furosemide are dehydration, blood volume
reduction, hypotension, electrolyte imbalance, hypokalemia and
hypochloremic alkalosis, and are extensions of its diuretic action.
 Option C: Metoprolol is not associated with this effect. Poisoning due to an
overdose of metoprolol may lead to severe hypotension, sinus bradycardia,
atrioventricular block, heart failure, cardiogenic shock, cardiac arrest,
bronchospasm, impairment of consciousness, coma, nausea, vomiting,
cyanosis, hypoglycaemia and, occasionally, hyperkalaemia.
Option D: This medication isn’t associated with such an effect. While there is

limited data about enalapril overdose in humans, overdosage may result in
marked hypotension and stupor based on the pharmacological properties of
the drug. Most common adverse effects of enalapril include cough,
hypotension, stupor, headache, dizziness and fatigue.
NCLEXRN-03-046

Question Tag: heart failure

Question Category: Physiological Integrity, Physiological Adaptation

Which of the following symptoms is most commonly associated with left-sided heart


failure?

 A. Crackles
 B. Arrhythmias
 C. Hepatic engorgement
 D. Hypotension
Correct Answer: A. Crackles

Crackles in the lungs are a classic sign of left-sided heart failure. These sounds are caused
by fluid backing up into the pulmonary system.  The left ventricle of the heart no longer
pumps enough blood around the body. As a result, blood builds up in the pulmonary veins
(the blood vessels that carry blood away from the lungs). This causes shortness of breath,
trouble breathing or coughing – especially during physical activity. Left-sided heart failure
is the most common type.

 Option B: Arrhythmias can be associated with both right and left-sided heart
failure. Cardiac arrhythmia is an abnormal rate and/or rhythm of a heart due to
its abnormal electrical impulse origination and/or propagation. Various
etiologies can cause arrhythmias. Heart failure (HF) is a clinical syndrome due
to an impaired heart that can not pump sufficient blood to meet the systemic
metabolic needs. 
 Option C: Hepatomegaly is prominent with chronic right-sided heart failure.
The primary pathophysiology involved in hepatic dysfunction from HF is either
passive congestion from increased filling pressures or low cardiac output and
the consequences of impaired perfusion. Passive hepatic congestion due to
increased central venous pressure may cause elevations of liver enzymes and
both direct and indirect serum bilirubin. 
 Option D: Left-sided heart failure causes hypertension secondary to an
increased workload on the system. Hypertension increases the workload on
the heart inducing structural and functional changes in the myocardium. These
changes include hypertrophy of the left ventricle, which can progress to heart
failure.
NCLEXRN-03-047

Question Tag: sacral edema

Question Category: Physiological Integrity, Physiological Adaptation

In which of the following disorders would the nurse expect to assess sacral edema in
bedridden client?

 A. DM
 B. Pulmonary emboli
 C. Renal failure
 D. Right-sided heart failure
Correct Answer: D. Right-sided heart failure

The most accurate area of the body to assess dependent edema in a bedridden client is the
sacral area. Sacral, or dependent, edema is secondary to right-sided heart failure.

Option A: Diabetes mellitus is not directly related to sacral edema. Insulin



edema is uncommon, and the extent of oedema may vary from limited ankle
swelling to severe.
 Option B: Pulmonary emboli does not cause sacral edema. Sacral edema
accumulates in the lower back (called sacral edema) after being in bed for
several hours. Pushing on the swollen area for a few seconds will leave a
temporary dimple or dent in the skin.
 Option C: Renal disease isn’t directly linked to sacral edema. When edema is
massive and generalized, it is called anasarca. It is caused by a variety of
clinical conditions like heart failure, renal failure, liver failure, or problems with
the lymphatic system. 
NCLEXRN-03-048

Question Tag: right-sided heart failure

Question Category: Physiological Integrity, Physiological Adaptation

Which of the following symptoms might a client with right-sided heart failure exhibit?

 A. Adequate urine output


 B. Polyuria
 C. Oliguria
 D. Polydipsia
Correct Answer: C. Oliguria

Inadequate deactivation of aldosterone by the liver after right-sided heart failure leads to
fluid retention, which causes oliguria. Oliguria is a late finding in heart failure, and it is
found in patients with markedly reduced cardiac output from severely reduced LV function.

Option A: An adequate urine output indicates that the client does not have

fluid retention. The activation of the RAAS leads to salt and water retention,
resulting in increased preload and further increases in myocardial energy
expenditure. This results in an increase in angiotensin II (Ang II) levels and, in
turn, aldosterone levels, causing stimulation of the release of aldosterone. Ang
II, along with ET-1, is crucial in maintaining effective intravascular homeostasis
as mediated by vasoconstriction and aldosterone-induced salt and water
retention.
 Option B: Polyuria is possible in a weak heart due to heart failure. A weak
heart cannot pump blood efficiently, causing fluids to build up in the body.
The kidneys work to rid the body of excess fluid, causing frequent urination.
 Option D: Polydipsia is usually associated with diabetes mellitus. In people
with diabetes, polydipsia is caused by increased blood glucose levels. When
blood glucose levels get high, the kidneys produce more urine in an effort to
remove the extra glucose from the body. Meanwhile, because the body is
losing fluids, the brain tells you to drink more in order to replace them.
NCLEXRN-03-049

Question Tag: ventricular contractility

Question Category: Physiological Integrity, Pharmacological and Parenteral Therapies

Which of the following classes of medications maximizes cardiac performance in clients


with heart failure by increasing ventricular contractility?

 A. Beta-adrenergic blockers
 B. Calcium channel blockers
 C. Diuretics
 D. Inotropic agents
Correct Answer: D. Inotropic agents

Inotropic agents are administered to increase the force of the heart’s contractions, thereby
increasing ventricular contractility and ultimately increasing cardiac output.
Option A: Beta blockers work by blocking the effects of the hormone

epinephrine, also known as adrenaline. They cause the heart to beat more
slowly and with less force, which lowers blood pressure.
 Option B: Calcium channel blockers decrease the heart rate and ultimately
decrease the workload of the heart.
 Option C: Diuretics are administered to decrease the overall vascular volume,
also decreasing the workload of the heart.
NCLEXRN-03-050

Question Tag: sympathetic nervous system

Question Category: Physiological Integrity, Physiological Adaptation

Stimulation of the sympathetic nervous system produces which of the following responses?

 A. Bradycardia
 B. Tachycardia
 C. Hypotension
 D. Decreased myocardial contractility
Correct Answer: B. Tachycardia

Stimulation of the sympathetic nervous system causes tachycardia and increased


contractility. Norepinephrine released from neurons via the right stellate ganglion increases
heart rate and shortens atrioventricular conduction via the sinus and atrioventricular nodes.

 Option A: The parasympathetic nervous system is responsible for decreasing


the heart rate. M1 receptors are the inhibitory receptors. They are present in
the heart, especially on SA node and AV node. Activation of M1 receptors
decreases heart rate. It does not have any effect on the force of contraction.
 Option C: Hypotension is caused by the parasympathetic nervous system. The
baroreceptor reflex stimulates the parasympathetic system. The PSNS causes
relaxation of blood vessels, decreasing total peripheral resistance. It also
decreases heart rate. As a result, the blood pressure decreases.
 Option D: Decreased myocardial contractility is related to the
parasympathetic nervous system, which is responsible for slowing the heart
rate.

NCLEXRN-03-051
Question Tag: heart failure

Question Category: Physiological Integrity, Physiological Adaptation

Which of the following conditions is most closely associated with weight gain, nausea, and
a decrease in urine output?

A. Angina pectoris
 B. Cardiomyopathy
 C. Left-sided heart failure
 D. Right-sided heart failure
Correct Answer: D. Right-sided heart failure

Weight gain, nausea, and a decrease in urine output are secondary effects of right-sided
heart failure. Here the right ventricle of the heart is too weak to pump enough blood to the
lungs. This causes blood to build up in the veins (the blood vessels that carry blood from
the organs and tissue back to the heart). The increased pressure inside the veins can push
fluid out of the veins into surrounding tissue. This leads to a build-up of fluid in the legs, or
less commonly in the genital area, organs or the abdomen (belly).

Option A: Angina pectoris doesn’t cause weight gain, nausea, or a decrease in



urine output. Angina is chest pain or discomfort caused when the heart muscle
doesn’t get enough oxygen-rich blood. It may feel like pressure or squeezing
in the chest. The discomfort also can occur in the shoulders, arms, neck, jaw, or
back. Angina pain may even feel like indigestion.
 Option B: Cardiomyopathy is usually identified as a symptom of left-sided
heart failure. The left ventricle of the heart no longer pumps enough blood
around the body. As a result, blood builds up in the pulmonary veins (the
blood vessels that carry blood away from the lungs). This causes shortness of
breath, trouble breathing or coughing – especially during physical activity.
Left-sided heart failure is the most common type.
 Option C: Left-sided heart failure causes primarily pulmonary symptoms
rather than systemic ones. Left-sided heart failure is usually caused by
coronary artery disease (CAD), a heart attack or long-term high blood
pressure.
NCLEXRN-03-052

Question Tag: abdominal aortic aneurysm

Question Category: Physiological Integrity, Physiological Adaptation

What is the most common cause of abdominal aortic aneurysm? 


 A. Atherosclerosis
 B. DM
 C. HPN
 D. Syphilis
Correct Answer: A. Atherosclerosis

Atherosclerosis accounts for 75% of all abdominal aortic aneurysms. Plaques build up on
the wall of the vessel and weaken it, causing an aneurysm. AAA is thought to be a
degenerative process of the aorta, the cause of which remains unclear. It is often attributed
to atherosclerosis because these changes are observed in the aneurysm at the time of
surgery.

Option B: Diabetes mellitus does not directly cause abdominal aortic



aneurysm. Patients at greatest risk for AAA are men who are older than 65
years and have peripheral atherosclerotic vascular disease. A history of
smoking often is elicited.
 Option C: Hypertension accounts for 1-15% cases of abdominal aneurysm. A
Swedish study showed that instances of AAA in elderly men have been
decreasing, A phenomenon that can be attributed to a nationwide decline in
smoking for the past 30 years, as well as the significantly improved longevity
of the elderly population.
 Option D: Syphilis is not related to AAA. Syphilis is a bacterial infection usually
spread by sexual contact. The disease starts as a painless sore — typically on
your genitals, rectum or mouth. Syphilis spreads from person to person via
skin or mucous membrane contact with these sores.
NCLEXRN-03-053

Question Tag: abdominal aortic aneurysm

Question Category: Physiological Integrity, Physiological Adaptation

In which of the following areas is an abdominal aortic aneurysm most commonly located?

 A. Distal to the iliac arteries


 B. Distal to the renal arteries
 C. Adjacent to the aortic branch
 D. Proximal to the renal arteries
Correct Answer: B. Distal to the renal arteries

The portion of the aorta distal to the renal arteries is more prone to an aneurysm because
the vessel isn’t surrounded by stable structures, unlike the proximal portion of the aorta.
Option A: Distal to the iliac arteries, the vessel is again surrounded by stable

vasculature, making this an uncommon site for an aneurysm.
 Option C: There is no area adjacent to the aortic arch, which bends into the
thoracic (descending) aorta.
 Option D: The proximal portion is surrounded by stable structures, lessening
the risk for rupture.
NCLEXRN-03-054

Question Tag: abdominal mass

Question Category: Physiological Integrity, Physiological Adaptation

A pulsating abdominal mass usually indicates which of the following conditions?

 A. Abdominal aortic aneurysm


 B. Enlarged spleen
 C. Gastric distention
 D. Gastritis
Correct Answer: A. Abdominal aortic aneurysm

The presence of a pulsating mass in the abdomen is an abnormal finding, usually indicating
an outpouching in a weakened vessel, as in abdominal aortic aneurysm. The finding,
however, can be normal for a thin person.

Option B: An enlarged spleen does not cause a pulsation. It is usually palpable

upon physical examination. Most clinically significant AAAs are palpable upon
routine physical examination; however, the sensitivity of palpation depends on
the experience of the examiner, the size of the aneurysm, and the size of the
patient. In one study, 38% of AAA cases were detected on the basis of physical
examination findings, whereas 62% were detected incidentally on radiologic
studies obtained for other reasons.
 Option C: Gastric distention does not cause pulsation. Gastric distension
causes progressive shortening of the abdominal length of the LES and a
reduction in its pressure. The process exposes the effaced mucosa and
sphincter to acid gastric juice.
 Option D: Gastritis does not indicate pulsation. The physical examination
findings are often normal, with occasional mild epigastric tenderness. The
examination tends to exhibit more abnormalities as the patient develops
complications in relation to gastritis.
NCLEXRN-03-055

Question Tag: abdominal aortic aneurysm


Question Category: Physiological Integrity, Physiological Adaptation

What is the most common symptom in a client with abdominal aortic aneurysm?

 A. Abdominal pain
 B. Diaphoresis
 C. Headache
 D. Upper back pain
Correct Answer: A. Abdominal pain

Abdominal pain in a client with an abdominal aortic aneurysm results from the disruption
of normal circulation in the abdominal region. Patients may experience unimpressive back,
flank, abdominal, or groin pain for some time before rupture. Isolated groin pain is a
particularly insidious presentation. This occurs with retroperitoneal expansion and pressure
on either the right or left femoral nerve. This symptom may be present without any other
associated findings, and a high index of suspicion is necessary to make the diagnosis.

Option B: Diaphoresis isn’t associated with abdominal aortic aneurysm. At



times, AAAs may cause symptoms from local compression, including early
satiety, nausea, vomiting, urinary symptoms, or venous thrombosis from
venous compression.
 Option C: Headache is not a sign of abdominal aortic aneurysm. Other
symptoms include abdominal pain, groin pain, embolic phenomena affecting
the toes (eg, livedo reticularis, or blue toe syndrome; see the image below),
and fever. Occasionally, small AAAs thrombose, producing acute claudication.
 Option D: Lower back pain, not upper, is a common symptom, usually
signifying expansion and impending rupture of the aneurysm. Back pain can
be caused by erosion of the AAA into adjacent vertebrae.
NCLEXRN-03-056

Question Tag: abdominal aortic aneurysm

Question Category: Physiological Integrity, Physiological Adaptation

Which of the following symptoms usually signifies rapid expansion and impending rupture
of an abdominal aortic aneurysm?

 A. Abdominal pain
 B. Absent pedal pulses
 C. Angina
 D. Lower back pain
Correct Answer: D. Lower back pain
Lower back pain results from expansion of an aneurysm. The expansion applies pressure in
the abdominal cavity, and the pain is referred to the lower back.

Option A: Abdominal pain is the most common symptom resulting from



impaired circulation. The most typical manifestation of rupture is abdominal or
back pain with a pulsatile abdominal mass. However, the symptoms may be
vague, and the abdominal mass may be missed.
 Option B: Absent pedal pulses are a sign of no circulation and would occur
after a ruptured aneurysm or in peripheral vascular disease.
 Option C: Angina is associated with atherosclerosis of the coronary arteries.
NCLEXRN-03-057

Question Tag: abdominal aortic aneurysm

Question Category: Physiological Integrity, Reduction of Risk Potential

What is the definitive test used to diagnose an abdominal aortic aneurysm?

 A. Abdominal X-ray
 B. Arteriogram
 C. CT scan
 D. Ultrasound
Correct Answer: B. Arteriogram

An arteriogram accurately and directly depicts the vasculature; therefore, it clearly


delineates the vessels and any abnormalities.

Option A: An abdominal aneurysm would only be visible on an X-ray if it were



calcified. Plain radiography is often performed on patients with abdominal
complaints before the diagnosis of AAA has been entertained. Using this
method to evaluate patients with AAA is difficult because the only marginally
specific finding, aortic wall calcification, is seen less than half of the time.
Aortic-wall calcification (see the images below) may appear without aneurysm
rim calcification, resulting in a high false-negative rate.
 Option C: CT scan doesn’t give a direct view of the vessels. CT permits
visualization of the retroperitoneum, is not limited by obesity or bowel gas,
detects leakage, and allows concomitant evaluation of the kidneys.
 Option D: Ultrasound doesn’t yield as accurate a diagnosis as the arteriogram.
Ultrasonography is the standard imaging tool for AAA. When performed by
trained personnel, it has a sensitivity of nearly 100% and a specificity
approaching 96% for the detection of infrarenal AAA.
NCLEXRN-03-058
Question Tag: abdominal aneurysm

Question Category: Physiological Integrity, Reduction of Risk Potential

Which of the following complications is of greatest concern when caring for a preoperative
abdominal aneurysm client?

A. HPN
 B. Aneurysm rupture
 C. Cardiac arrhythmias
 D. Diminished pedal pulses
Correct Answer: B. Aneurysm rupture

Rupture of an aneurysm is a life-threatening emergency and is of the greatest concern for


the nurse caring for this type of client. The layers of the aortic wall can also separate (aortic
dissection). This produces severe, tearing pain in the chest, back or abdomen. The potential
for rupture is the most serious risk associated with an aortic aneurysm. A ruptured aortic
aneurysm can cause life-threatening internal bleeding and/or a stroke.

Option A: Hypertension should be avoided and controlled because it can



cause the weakened vessel to rupture. Hypertension has been considered a
potential risk factor for AAA; but the findings from prospective cohort studies
have not been entirely consistent, nor have they been summarised in a
comprehensive meta-analysis.
 Option D: Diminished pedal pulses, a sign of poor circulation to the lower
extremities, are associated with an aneurysm but aren’t life-threatening. The
appearance of microembolic lower limb infarcts in a patient with easily
palpable pedal pulses may suggest the presence of either popliteal or
abdominal aneurysms.
 Option C: Cardiac arrhythmias aren’t directly linked to an aneurysm.
Ventricular aneurysms may cause shortness of breath, chest pain, or an
irregular heart beat (arrhythmia).
NCLEXRN-03-059

Question Tag: aneurysm

Question Category: Physiological Integrity, Physiological Adaptation

Which of the following blood vessel layers may be damaged in a client with an aneurysm?

 A. Externa
 B. Interna
 C. Media
 D. Interna and Media
Correct Answer: C. Media

The factor common to all types of aneurysms is a damaged media. The media has more
smooth muscle and less elastic fibers, so it’s more capable of vasoconstriction and
vasodilation.

Option A: The tunica externa generally has no damage in an aneurysm. The



outermost layer, tunica externa, comprises connective tissue providing
protection for the vessel. 
 Option B: The interna generally not damaged in an aneurysm. The tunica
intima (New Latin “inner coat”), or intima for short, is the innermost tunica
(layer) of an artery or vein. It is made up of one layer of endothelial cells and is
supported by an internal elastic lamina. The endothelial cells are in direct
contact with the blood flow.
 Option D: The media is the most damaged among the blood vessel layers.
The tunica media is composed of smooth muscle cells and elastic tissue that is
responsible for autoregulation of cerebral blood flow.
NCLEXRN-03-060

Question Tag: abdominal aortic aneurysm

Question Category: Health Promotion and Maintenance

When assessing a client for an abdominal aortic aneurysm, which area of the abdomen is
most commonly palpated?

 A. Right upper quadrant


 B. Directly over the umbilicus
 C. Middle lower abdomen to the left of the midline
 D. Midline lower abdomen to the right of the midline
Correct Answer: C. Middle lower abdomen to the left of the midline

The aorta lies directly left of the umbilicus; therefore, any other region is inappropriate for
palpation. The aortic pulse can be palpated just above and to the left of the umbilicus. The
width of the aorta can then be measured by placing both hands palms down on the
patient’s abdomen, with one index finger on either side of the aorta. Each systole should
move the fingers apart.

 Option A: The right upper quadrant is a little farther to the appropriate area
to be palpated. Abdominal examination includes palpation of the aorta and
estimation of the size of the aneurysm. AAAs are palpated in the upper
abdomen; the aorta bifurcates into the iliac arteries just above the umbilicus.
Option B: Palpating the umbilicus would yield unappropriate results. The

examiner may palpate the periumbilical area for any defect, mass, or
an umbilical hernia. The patient can be asked to cough or bear down to feel
for any protruding mass. 
 Option D: The aorta lies to the left, not the right, of the umbilicus.  Left lower
quadrant tenderness may be a presenting sign of diverticulitis in the elderly. A
mass, if present may be due to a tumor of the colon, a left ovarian cyst,
or ectopic pregnancy. In the elderly, constipation leading to impacted feces
may also present with a mass palpated in the left lower quadrant.
NCLEXRN-03-061

Question Tag: aneurysm

Question Category: Health Promotion and Maintenance

Which of the following conditions is linked to more than 50% of clients with abdominal
aortic aneurysms?

 A. DM
 B. HPN
 C. PVD
 D. Syphilis
Correct Answer: B. HPN

Continuous pressure on the vessel walls from hypertension causes the walls to weaken and
an aneurysm to occur. The association between hypertension and AAA could potentially be
confounded by other risk factors because hypertension is more common among persons
with overweight and obesity, less physical activity, who smoke and who have unhealthy
diets.

 Option A: Diabetes mellitus doesn’t have a direct link to aneurysm. Diabetes


mellitus (DM) is a strong cardiovascular risk factor; however, multiple
epidemiological studies have confirmed that a negative relationship exists
between DM and abdominal aortic aneurysm (AAA) presence, growth and
rupture.  Arteries from patients with DM are often harder and more calcified
than those from patients without DM; however, increased vessel wall
calcification alone does not appear to explain the reduced rate of aortic
expansion seen in diabetic patients.
 Option C: Atherosclerotic changes can occur with peripheral vascular diseases
and are linked to aneurysms, but the link isn’t as strong as it is with
hypertension.
 Option D: Only 1% of clients with syphilis experience an aneurysm. Classically,
syphilitic aneurysms occur in 90% of cases on the thoracic aorta, and in 10% in
the abdominal aorta [3, 7–9]. Infection of the aortic wall develops during the
secondary or bacteraemic phase of syphilis, having a latent period from
infection until the clinical presentation ranging from 5 to 50 years.
NCLEXRN-03-062

Question Tag: auscultation

Question Category: Health Promotion and Maintenance

Which of the following sounds is distinctly heard on auscultation over the abdominal
region of an abdominal aortic aneurysm client?

 A. Bruit
 B. Crackles
 C. Dullness
 D. Friction rubs
Correct Answer: A. Bruit

A bruit, a vascular sound resembling heart murmur, suggests partial arterial occlusion. In
addition to abdominal palpation, auscultation for abdominal or femoral bruits may be
useful for clinical detection of AAA. Auscultation is performed along the course of the
aortic and femoral arteries. However, absence of a bruit does not exclude an aneurysm.

Option B: Crackles are indicative of fluid in the lungs. Crackles occur if the

small air sacs in the lungs fill with fluid and there’s any air movement in the
sacs, such as when breathing. The air sacs fill with fluid when a person has
pneumonia or heart failure. 
 Option C: Dullness is heard over solid organs, such as the liver. Dull or thud
like sounds are normally heard over dense areas such as the heart or liver.
Dullness replaces resonance when fluid or solid tissue replaces air-containing
lung tissues, such as occurs with pneumonia, pleural effusions, or tumors
 Option D: Friction rubs indicate inflammation of the peritoneal surface. A
pericardial friction rub is pathognomonic for acute pericarditis; the rub has a
scratching, grating sound similar to leather rubbing against leather. Serial
examinations may be necessary for detection, as a friction rub may be
transient from one hour to the next and is present in approximately 50% of
cases.
NCLEXRN-03-063

Question Tag: ruptured abdominal aneurysm

Question Category: Physiological Integrity, Physiological Adaptation


Which of the following groups of symptoms indicated a ruptured abdominal aneurysm?

 A. Lower back pain, increased BP, decreased RBC, increased WBC


 B. Severe lower back pain, decreased BP, decreased RBC, increased WBC
 C. Severe lower back pain, decreased BP, decreased RBC, decreased WBC
 D. Intermittent lower back pain, decreased BP, decreased RBC, increased WBC
Correct Answer: B. Severe lower back pain, decreased BP, decreased RBC, increased
WBC

Severe lower back pain indicates an aneurysm rupture, secondary to pressure being
applied within the abdominal cavity. When rupture occurs, the pain is constant because it
can’t be alleviated until the aneurysm is repaired. Blood pressure decreases due to the loss
of blood. After the aneurysm ruptures, the vasculature is interrupted and blood volume is
lost, so blood pressure wouldn’t increase. For the same reason, the RBC count has
decreased – not increased. The WBC count increases as cells migrate to the site of injury.

Option A: The pain felt during rupture is severe. Due to the loss of blood, the

blood pressure decreases. 
 Option C: The WBC count increases because the cells migrate to the site of
injury.
 Option D: The pain is not intermittent during an aneurysm; it is constant and
severe.
NCLEXRN-03-064

Question Tag: hematoma

Question Category: Physiological Integrity, Reduction of Risk Potential

Which of the following complications of an abdominal aortic repair is indicated by


detection of a hematoma in the perineal area?

 A. Hernia
 B. Stage 1 pressure ulcer
 C. Retroperitoneal rupture at the repair site
 D. Rapid expansion of the aneurysm
Correct Answer: C. Retroperitoneal rupture at the repair site

Blood collects in the retroperitoneal space and is exhibited as a hematoma in the perineal
area. This rupture is most commonly caused by leakage at the repair site.

 Option A: A hernia doesn’t cause vascular disturbances. A hernia is the


abnormal exit of tissue or an organ, such as the bowel, through the wall of the
cavity in which it normally resides. Hernias come in a number of types. Most
commonly they involve the abdomen, specifically the groin. Groin hernias are
most commonly of the inguinal type but may also be femoral.
 Option B: A pressure ulcer does not cause significant bleeding, and does not
cause a hematoma. Bedsores — also called pressure ulcers and decubitus
ulcers — are injuries to skin and underlying tissue resulting from prolonged
pressure on the skin. Bedsores most often develop on skin that covers bony
areas of the body, such as the heels, ankles, hips and tailbone.
 Option D: Because no bleeding occurs with rapid expansion of the aneurysm,
a hematoma won’t form. Fast growth of abdominal aortic aneurysm (AAA)
diameter is claimed to be an indication for aneurysm repair. If fast growth is a
valid indication for operative repair then an episode of measured fast growth
should be followed by sustained rapid expansion and a high risk of rupture.
NCLEXRN-03-065

Question Tag: aneurysm

Question Category: Physiological Integrity, Physiological Adaptation

Which hereditary disease is most closely linked to aneurysm?

 A. Cystic fibrosis
 B. Lupus erythematosus
 C. Marfan’s syndrome
 D. Myocardial infarction
Correct Answer: C. Marfan’s syndrome

Marfan’s syndrome results in the degeneration of the elastic fibers of the aortic media.
Therefore, clients with the syndrome are more likely to develop an aortic aneurysm. Marfan
syndrome (MFS) is a spectrum of disorders caused by a heritable genetic defect of
connective tissue that has an autosomal dominant mode of transmission. The defect itself
has been isolated to the FBN1 gene on chromosome 15, which codes for the connective
tissue protein fibrillin. Abnormalities in this protein cause a myriad of distinct clinical
problems, of which the musculoskeletal, cardiac, and ocular system problems predominate.

 Option A: Although cystic fibrosis is hereditary, it hasn’t been linked to


aneurysms. Cystic fibrosis (CF) is the most common potentially lethal genetic
disease in the white population. Improvements in life expectancy have led to
an increasing recognition of hepatobiliary complications from CF. Splenic
artery aneurysms are a rare complication of portal hypertension with a high
mortality due to their significant potential for rupture, resulting in life-
threatening i.p. haemorrhage.
Option B: Lupus erythematosus isn’t hereditary. Systemic lupus

erythematosus (SLE), is the most common type of lupus. SLE is an autoimmune
disease in which the immune system attacks its own tissues, causing
widespread inflammation and tissue damage in the affected organs. It can
affect the joints, skin, brain, lungs, kidneys, and blood vessels.
 Option D: Myocardial infarction is neither hereditary nor a disease. Myocardial
infarction (MI) (ie, heart attack) is the irreversible death (necrosis) of heart
muscle secondary to prolonged lack of oxygen supply (ischemia).
Approximately 1.5 million cases of MI occur annually in the United States.
NCLEXRN-03-066

Question Tag: ruptured aneurysm

Question Category: Physiological Integrity, Physiological Adaptation

Which of the following treatments is the definitive one for a ruptured aneurysm?

 A. Antihypertensive medication administration


 B. Aortogram
 C. Beta-adrenergic blocker administration
 D. Surgical intervention
Correct Answer: D. Surgical intervention

When the vessel ruptures, surgery is the only intervention that can repair it. The techniques
used for aneurysm surgery have for long been standardized. 95% of aneurysms are
accessible via a small frontotemporal craniotomy centred over the pterion. Only occasional
cases, principally aneurysms of the distal anterior cerebral artery and the lower
vertebrobasilar trunk, require different surgical approaches.

 Option A: Administration of antihypertensive medications can help control


hypertension, reducing the risk of rupture. Normalization of blood pressure by
hydralazine significantly reduced the incidence of ruptured aneurysms and the
rupture rate. There was a dose dependent relationship between the reduction
of blood pressure and the prevention of aneurysmal rupture. Captopril and
losartan were able to reduce the rupture rates without affecting systemic
hypertension induced by DOCA-salt treatment.
 Option B: An aortogram is a diagnostic tool used to detect an aneurysm. An
aortogram is an invasive diagnostic test using a catheter to inject dye (contrast
medium) into the aorta. X-rays are taken of the dye as it travels within the
aorta, allowing clear visualization of blood flow. This way, any structural
abnormalities of the aorta will be accurately seen.
 Option C: Beta blockers can slow the heartbeat, thereby decreasing blood
pressure. Antihypertensives are used to reduce the rate of rise of the aortic
pressure (dP/dt). For acute reduction of arterial pressure, the potent
vasodilator sodium nitroprusside is very effective. To reduce dP/dt acutely,
administer a beta blocker intravenously (IV) in incremental doses until a heart
rate of 60-80 beats/min is attained. When beta blockers are contraindicated,
as in second- or third-degree atrioventricular block, consider using calcium-
channel blockers.
NCLEXRN-03-067

Question Tag: cardiovascular diseases

Question Category: Physiological Integrity, Reduction of Risk Potential

Which of the following heart muscle diseases is unrelated to other cardiovascular diseases?

 A. Cardiomyopathy
 B. Coronary artery disease
 C. Myocardial infarction
 D. Pericardial Effusion
Correct Answer: A. Cardiomyopathy

Cardiomyopathy isn’t usually related to an underlying heart disease such as atherosclerosis.


The etiology in most cases is unknown.

Option B: Coronary artery disease is directly related to atherosclerosis.



Coronary artery disease is caused by plaque buildup in the wall of the arteries
that supply blood to the heart (called coronary arteries). Plaque is made up of
cholesterol deposits. Plaque buildup causes the inside of the arteries to narrow
over time. This process is called atherosclerosis.
 Option C: During progression of atherosclerosis, myeloid cells destabilize
lipid-rich plaque in the arterial wall and cause its rupture, thus triggering
myocardial infarction.
 Option D: Pericardial effusion is the escape of fluid into the pericardial sac, a
condition associated with pericarditis and advanced heart failure.
NCLEXRN-03-068

Question Tag: cardiomyopathy

Question Category: Physiological Integrity, Physiological Adaptation

Which of the following types of cardiomyopathy can be associated with childbirth?

 A. Dilated
 B. Hypertrophic
 C. Myocarditis
 D. Restrictive
Correct Answer: A. Dilated

Although the cause isn’t entirely known, cardiac dilation and heart failure may develop
during the last month of pregnancy or the first few months after birth. The condition may
result from a pre existing cardiomyopathy not apparent prior to pregnancy.

Option B: Hypertrophic cardiomyopathy is an abnormal symmetry of the



ventricles that has an unknown etiology but a strong familial tendency.
 Option C: Myocarditis isn’t specifically associated with childbirth. Myocarditis
is an inflammation of the heart muscle (myocardium). Myocarditis can affect
the heart muscle and the heart’s electrical system, reducing the heart’s ability
to pump and causing rapid or abnormal heart rhythms (arrhythmias).
 Option D: Restrictive cardiomyopathy indicates constrictive pericarditis; the
underlying cause is usually myocardial. Restrictive cardiomyopathy (RCM) is a
rare disease of the myocardium and is the least common of the three clinically
recognized and described cardiomyopathies. It is characterized by diastolic
dysfunction with restrictive ventricular physiology, whereas systolic function
often remains normal. Atrial enlargement occurs due to impaired ventricular
filling during diastole, but the volume and wall thickness of the ventricles are
usually normal.
NCLEXRN-03-069

Question Tag: cardiomyopathy

Question Category: Physiological Integrity, Physiological Adaptation

Septal involvement occurs in which type of cardiomyopathy?

 A. Congestive
 B. Dilated
 C. Hypertrophic
 D. Restrictive
Correct Answer: C. Hypertrophic

In hypertrophic cardiomyopathy, hypertrophy of the ventricular septum – not the ventricle


chambers – is apparent. Hypertrophic cardiomyopathy (HCM) is a genetic cardiovascular
disease. It is defined by an increase in left ventricular wall thickness that is not solely
explained by abnormal loading conditions. This disorder is caused by a mutation in cardiac
sarcomere protein genes and is most frequently transmitted as an autosomal dominant
trait.
Option A: In congestive cardiomyopathy, the heart becomes stretched and

weakened and is unable to pump effectively. Congestive cardiomyopathy is a
clinical state in which an abnormality of ventricular myocardium results in
impaired pump function and circulatory congestion.
 Option B: Dilated cardiomyopathy is also called congestive cardiomyopathy,
which is a condition wherein the heart cannot pump effectively due to its
weakening state. In dilated cardiomyopathy, the heart’s ability to pump blood
is decreased because the heart’s main pumping chamber, the left ventricle, is
enlarged, dilated and weak. At first, the chambers of the heart respond by
stretching to hold more blood to pump through the body. This helps to
strengthen the heart’s contraction and keep the blood moving for a short
while. With time, the heart muscle walls weaken and are not able to pump as
strongly.
  Option D: Restrictive cardiomyopathy, the rarest form of cardiomyopathy, is a
condition in which the walls of the lower chambers of the heart (the ventricles)
are abnormally rigid and lack the flexibility to expand as the ventricles fill with
blood.
NCLEXRN-03-070

Question Tag: cardiomyopathy

Question Category: Physiological Integrity, Physiological Adaptation

Which of the following recurring conditions most commonly occurs in clients with


cardiomyopathy?

 A. Heart failure
 B. DM
 C. MI
 D. Pericardial effusion
Correct Answer: A. Heart failure

Because the structure and function of the heart muscle is affected, heart failure most
commonly occurs in clients with cardiomyopathy. Heart failure can occur when the heart
muscle is weak (systolic failure) or when it is stiff and unable to relax normally (diastolic
failure). Cardiomyopathy, which means “disease of the heart muscle,” is one of many
causes of heart failure.

 Option B: Diabetes mellitus is a group of metabolic disease characterized by


hyperglycemia resulting from defects in insulin secretion, insulin action, or
both. Type 1 and type 2 diabetic patients are at increased risk of
cardiomyopathy and heart failure is a major cause of death for these patients.
Cardiomyopathy in diabetes is associated with a cluster of features including
decreased diastolic compliance, interstitial fibrosis and myocyte hypertrophy.
 Option C: Myocardial infarction results from prolonged myocardial ischemia
due to reduced blood flow through one of the coronary arteries.
 Option D: Pericardial effusion is most predominant in clients with pericarditis.
Diabetes mellitus is unrelated to cardiomyopathy. Pericardial effusion puts
pressure on the heart, affecting the heart’s function. If untreated, it can lead to
heart failure or death.
NCLEXRN-03-071

Question Tag: heart muscle

Question Category: Physiological Integrity, Physiological Adaptation

What is the term used to describe an enlargement of the heart muscle?

 A. Cardiomegaly
 B. Cardiomyopathy
 C. Myocarditis
 D. Pericarditis
Correct Answer: A. Cardiomegaly

Cardiomegaly denotes an enlarged heart muscle. The most critical pathophysiological


changes leading to cardiomegaly include dilated hypertrophy, fibrosis, and contractile
malfunction. Contractile dysfunction and abnormal myocardial remodeling can lead to
hypertrophic cardiomyopathy or dilated cardiomyopathy. Mechanical stretching, circulating
neurohormones and oxidative stress are significant stimuli for the signal transduction of
inflammatory cytokines and MAP kinase in cardiomyocytes. Signal transduction leads to
changes in structural proteins and proteins that regulate excitation-contraction. Dilated
cardiomyopathy mutations result in a reduced force of the sarcomere contraction and a
reduction in sarcomere content. Hypertrophic cardiomyopathy mutations result in a
molecular phenotype of hyperdynamic contractility, poor relaxation, and increased energy
consumption.

 Option B: Cardiomyopathy is a heart muscle disease of unknown origin. In


cardiomyopathy, the heart muscle becomes enlarged, thick, or rigid. In rare
cases, the muscle tissue in the heart is replaced with scar tissue.
 Option C: Myocarditis refers to inflammation of heart muscle. It is an
inflammatory disease of the myocardium with a wide range of clinical
presentations, from subtle to devastating.
 Option D: Pericarditis is an inflammation of the pericardium. Pericarditis is
usually acute – it develops suddenly and may last up to several months. The
condition usually clears up after 3 months, but sometimes attacks can come
and go for years. When a client has pericarditis, the membrane around the
heart is red and swollen, like the skin around a cut that becomes inflamed.
Sometimes there is extra fluid in the space between the pericardial layers,
which is called pericardial effusion.
NCLEXRN-03-072

Question Tag: heart failure

Question Category: Physiological Integrity, Physiological Adaptation

Dyspnea, cough, expectoration, weakness, and edema are classic signs and symptoms of
which of the following conditions?

 A. Pericarditis
 B. Hypertension
 C. Obliterative
 D. Restrictive
Correct Answer: D. Restrictive

These are the classic symptoms of heart failure. Heart failure is the pathophysiologic state
in which the heart, via an abnormality of cardiac function (detectable or not), fails to pump
blood at a rate commensurate with the requirements of the metabolizing tissues or is able
to do so only with an elevated diastolic filling pressure.

Option A: Pericarditis is exhibited by a feeling of fullness in the chest and



auscultation of a pericardial friction rub.
 Option B: Hypertension is usually exhibited by headaches, visual disturbances,
and a flushed face. Myocardial infarction causes heart failure but isn’t related
to these symptoms.
 Option C: Obliterative cardiomyopathy is very rare. It may result from the end
stage of eosinophilic syndromes, in which intracavitary thrombus fills the left
ventricular apex and hampers the filling of the ventricles.
NCLEXRN-03-073

Question Tag: cardiomyopathy

Question Category: Physiological Integrity, Physiological Adaptation

Which of the following types of cardiomyopathy does not affect cardiac output?

 A. Dilated
 B. Hypertrophic
C. Restrictive
 D. Obliterative
Correct Answer: B. Hypertrophic

Cardiac output isn’t affected by hypertrophic cardiomyopathy because the size of the
ventricle remains relatively unchanged. Three explanations for the systolic anterior motion
of the mitral valve have been offered, as follows: (1) the mitral valve is pulled against the
septum by contraction of the papillary muscles, which occurs because of the valve’s
abnormal location and septal hypertrophy altering the orientation of the papillary muscles;
(2) the mitral valve is pushed against the septum because of its abnormal position in the
outflow tract; (3) the mitral valve is drawn toward the septum because of the lower
pressure that occurs as blood is ejected at high velocity through a narrowed outflow tract
(Venturi effect).

Option A: Dilated cardiomyopathy causes a decrease in cardiac output.



Progressive dilation can lead to significant mitral and tricuspid regurgitation,
which may further diminish the cardiac output and increase end-systolic
volumes and ventricular wall stress. In turn, this leads to further dilation and
myocardial dysfunction.
 Option C: Restrictive cardiomyopathy causes decreased cardiac output.
Reduced LV filling leads to reduced stroke volume resulting in low cardiac
output symptoms such as fatigue and lethargy. Increased filling pressures
cause pulmonary and systemic congestion and symptomatic dyspnea.
 Option D: Obliterative cardiomyopathy may affect cardiac output because a
thrombus hampers ventricular filling, which may decrease the cardiac output.
NCLEXRN-03-074

Question Tag: heart sounds

Question Category: Physiological Integrity, Physiological Adaptation

Which of the following cardiac conditions does a fourth heart sound (S4) indicate?

A. Dilated aorta


 B. Normally functioning heart
 C. Decreased myocardial contractility
 D. Failure of the ventricle to eject all the blood during systole
Correct Answer: D. Failure of the ventricle to eject all the blood during systole

An S4 occurs as a result of increased resistance to ventricular filling after atrial contraction.


This increased resistance is related to decreased compliance of the ventricle.
Option A: A dilated aorta doesn’t cause an extra heart sound, though it does

cause a murmur. The aorta is considered pathologically dilated if the
diameters of the ascending aorta and the aortic root exceed the norms for a
given age and body size. A 50% increase over the normal diameter is
considered aneurysmal dilatation.
 Option B: An s4 isn’t heard in a normally functioning heart. The fourth heart
sound is a low-pitched sound coincident with late diastolic filling of the
ventricle due to atrial contraction. It thus occurs shortly before the first heart
sound. Although it is also called the atrial sound, and its production requires
an effective atrial contraction, the fourth heart sound is the result of vibrations
generated within the ventricle.
 Option C: Decreased myocardial contractility is heard as a third heart sound.
Optimal myocardial contractility is dependent on an optimal filling pressure,
afterload, and the presence and availability of inotropic substances (eg,
epinephrine, norepinephrine, or calcium). Calcium influx and binding to
troponin C is essential for cardiac contraction.
NCLEXRN-03-075

Question Tag: cardiomyopathy

Question Category: Pharmacological and Parenteral Therapies

Which of the following classes of drugs is most widely used in the treatment of


cardiomyopathy?

 A. Antihypertensive
 B. Beta-adrenergic blockers
 C. Calcium channel blockers
 D. Nitrates
Correct Answer: B. Beta-adrenergic blockers

By decreasing the heart rate and contractility, beta-adrenergic blockers improve myocardial
filling and cardiac output, which are primary goals in the treatment of cardiomyopathy.

 Option A: Antihypertensives aren’t usually indicated because they would


decrease cardiac output in clients who are often already hypotensive. Many
antihypertensive drugs have their primary action on systemic vascular
resistance. Some of these drugs produce vasodilation by interfering with
sympathetic adrenergic vascular tone (sympatholytics) or by blocking the
formation of angiotensin II or its vascular receptors.
 Option C: Calcium channel blockers are sometimes used for the same reasons
as beta-adrenergic blockers; however, they aren’t as effective as beta-
adrenergic blockers and cause increased hypotension. These channels are
responsible for regulating the influx of calcium into muscle cells, which in turn
stimulates smooth muscle contraction and cardiac myocyte contraction. In
cardiac nodal tissue, L-type calcium channels play an important role in and in
phase 0 of the action potentials. 
 Option D: Nitrates aren’t used because of their dilating effects, which would
further compromise the myocardium. Nitrates exert their effects by dilating
venous vessels, coronary arteries, and small arterioles; its maximal vasodilation
is in the venous vessels.

You might also like